emacs-orgmode@gnu.org archives
 help / color / mirror / code / Atom feed
* Bug: cannot export to beamer, even with (require 'ox-beamer) [9.1.14 (release_9.1.14-1059-gadec50 @ /home/oub/emacs/site-lisp/packages/org/)]
@ 2018-12-17  8:07 Uwe Brauer
  2018-12-17 18:07 ` Nick Dokos
  0 siblings, 1 reply; 5+ messages in thread
From: Uwe Brauer @ 2018-12-17  8:07 UTC (permalink / raw)
  To: emacs-orgmode


Hi

Some time ago I added via customize the exam class to my setting.
So the relevant custom entry looks like this 

(custom-set-variables
 '(org-latex-classes
   '(("article" "\\documentclass[12pt]{article}"
      ("\\section{%s}" . "\\section*{%s}")
      ("\\subsection{%s}" . "\\subsection*{%s}")
      ("\\subsubsection{%s}" . "\\subsubsection*{%s}")
      ("\\paragraph{%s}" . "\\paragraph*{%s}")
      ("\\subparagraph{%s}" . "\\subparagraph*{%s}"))
     ("report" "\\documentclass[12pt]{report}"
      ("\\part{%s}" . "\\part*{%s}")
      ("\\chapter{%s}" . "\\chapter*{%s}")
      ("\\section{%s}" . "\\section*{%s}")
      ("\\subsection{%s}" . "\\subsection*{%s}")
      ("\\subsubsection{%s}" . "\\subsubsection*{%s}"))
     ("book" "\\documentclass[12pt]{book}"
      ("\\part{%s}" . "\\part*{%s}")
      ("\\chapter{%s}" . "\\chapter*{%s}")
      ("\\section{%s}" . "\\section*{%s}")
      ("\\subsection{%s}" . "\\subsection*{%s}")
      ("\\subsubsection{%s}" . "\\subsubsection*{%s}"))
     ("exam" "\\documentclass[12pt, addpoints, answers]{exam}"
      ("\\begin{questions}%%%s" "\\end{questions}" "\\begin{questions}%%%s" "\\end{questions}")
      ("\\titledquestion{%s}" . "")
      ("\\begin{parts}%%%s" "\\end{parts}" "\\begin{parts}%%%s" "\\end{parts}")
      ("\\part[%s] " . "")
      ("\\begin{subparts}%%%s" "\\end{subparts}" "\\begin{subparts}%%%s" "\\end{subparts}")
      ("\\subpart[%s] " . "")
      ("\\begin{solution}[%s]" "\\end{solution}" "\\begin{solution}[%s]" "\\end{solution}"))))

I have  
(require 'ox-beamer) 

In my org init file, which should via add-to-list, add beamer but somehow
it does not. 

So when I call 

C-c C-e l b

In a org file I obtain the message

user-error: Unknown LaTeX class ‘beamer’

Something is wrong here. Add-to-list should add beamer to the org-class
but somehow it does not.
I am not sure who is to blame. Org or customize?

Uwe Brauer 



Emacs  : GNU Emacs 27.0.50 (build 1, x86_64-pc-linux-gnu, X toolkit, Xaw3d scroll bars)
 of 2018-04-26
Package: Org mode version 9.1.14 (release_9.1.14-1059-gadec50 @ /home/oub/emacs/site-lisp/packages/org/)

current state:
==============
(setq
 org-table-export-default-format "my-tbl-to-csv"
 org-src-mode-hook '(org-src-babel-configure-edit-buffer org-src-mode-configure-edit-buffer)
 org-after-todo-state-change-hook '(org-clock-out-if-current)
 org-latex-listings t
 org-capture-prepare-finalize-hook '(gnorb-registry-capture gnorb-org-capture-abort-cleanup)
 org-blank-before-new-entry '((plain-list-item . t))
 org-babel-after-execute-hook '(org-display-inline-images)
 org-metadown-hook '(org-babel-pop-to-session-maybe)
 org-clock-out-hook '(org-clock-remove-empty-clock-drawer)
 org-latex-to-mathml-jar-file "/home/oub/ALLES/src/MathWeb/mathtoweb.jar"
 org-html-format-inlinetask-function 'org-html-format-inlinetask-default-function
 org-time-stamp-custom-formats '(" %d.%m.%y " . " %d.%m.%y %a %H:%M ")
 org-pretty-entities t
 org-enforce-todo-dependencies t
 org-ref-create-notes-hook '((lambda nil (org-narrow-to-subtree) (insert (format "cite:%s\n" (org-entry-get (point) "Custom_ID")))))
 org-odt-format-headline-function 'org-odt-format-headline-default-function
 org-agenda-files '("~/ALLES/HGs/JCR-Science-Citation-Index/JCR-5years-2017-math.org" "~/tex/Babba/Beihilfe-Debeka/Beihilfe-Debka-gesamt.org"
                    "~/ALLES/HGs/Karp/Pub/Letter.org" "~/ALLES/HGs/tex/CitationIndex/Mat-2016.org" "~/ALLES/MathRev/ToDo/TODO.org"
                    "~/ALLES/HGs/tex/vorlesungen/HGQuim-Num-Estad/Klausur/2017-18/Jun/examn-jun18.org" "~/Mail/Remember/remember.org"
                    "~/ALLES/Mail/README.org")
 org-ascii-format-inlinetask-function 'org-ascii-format-inlinetask-default
 org-latex-to-mathml-convert-command "java -jar %j -unicode -force -df %o %I"
 org-blocker-hook '(org-block-todo-from-children-or-siblings-or-parent)
 org-gnus-prefer-web-links t
 org-calc-default-modes '(calc-internal-prec 12 calc-float-format (float 4) calc-angle-mode deg calc-prefer-frac nil calc-symbolic-mode nil
                          calc-date-format (YYYY "-" MM "-" DD " " Www (" " hh ":" mm)) calc-display-working-message t)
 org-format-latex-header "\\documentclass{article}\n\\usepackage[usenames]{color}\n[PACKAGES]\n[DEFAULT-PACKAGES]\n\\pagestyle{empty}             % do not remove\n% The settings below are copied from fullpage.sty\n\\setlength{\\textwidth}{\\paperwidth}\n\\usepackage{amsmath,amssymb}\n\\addtolength{\\textwidth}{-3cm}\n\\setlength{\\oddsidemargin}{1.5cm}\n\\addtolength{\\oddsidemargin}{-2.54cm}\n\\setlength{\\evensidemargin}{\\oddsidemargin}\n\\setlength{\\textheight}{\\paperheight}\n\\addtolength{\\textheight}{-\\headheight}\n\\addtolength{\\textheight}{-\\headsep}\n\\addtolength{\\textheight}{-\\footskip}\n\\addtolength{\\textheight}{-3cm}\n\\setlength{\\topmargin}{1.5cm}\n\\newtheorem{thm}{Theorem}\n\\newtheorem{defn}{Definition}\n\\newtheorem{lem}{Lemma}\n\\newtheorem{cor}{Corollary}\n\\newtheorem{rem}{Remark}\n\\newtheorem{bei}{}\n\\newtheorem{prop}{Preposition}\n\\newcommand{\\setR}{{\\mathord{\\mathbb R}}}\n\\addtolength{\\topmargin}{-2.54cm} "
 org-mode-hook '(#[0 "\301\211\207" [imenu-create-index-function org-imenu-get-tree] 2]
                 (lambda nil (local-set-key [C-M-up] 'org-table-move-single-cell-up)
                  (local-set-key [C-M-down] 'org-table-move-single-cell-down) (local-set-key [C-M-left] 'org-table-move-single-cell-left)
                  (local-set-key [C-M-right] 'org-table-move-single-cell-right))
                 org-ref-org-menu ob-ipython-auto-configure-kernels turn-on-diff-hl-mode turn-on-auto-fill my-org-keys turn-on-flyspell
                 turn-on-auto-capitalize-mode org-tempo-setup org-auctex-keys-minor-mode navi-make-org-mode-promotion-headings-list
                 #[0 "\300\301\302\303\304$\207" [add-hook change-major-mode-hook org-show-all append local] 5]
                 #[0 "\300\301\302\303\304$\207" [add-hook change-major-mode-hook org-babel-show-result-all append local] 5]
                 org-babel-result-hide-spec org-babel-hide-all-hashes)
 org-odt-format-drawer-function #[514 "\207" [] 3 "\n\n(fn NAME CONTENTS)"]
 org-archive-hook '(org-attach-archive-delete-maybe)
 org-confirm-elisp-link-function 'yes-or-no-p
 org-agenda-finalize-hook '(gnorb-org-popup-bbdb org-agenda-property-add-properties)
 org-ref-bibliography-entry-format '(("misc" . "%a %t <i>%h</i> (%y) <a href=\"%U\">link</a>")
                                     ("article" .
                                      "%a, %t, <i>%j</i>, <b>%v(%n)</b>, %p (%y). <a href=\"%U\">link</a>. <a href=\"http://dx.doi.org/%D\">doi</a>. <b>%N</b>")
                                     ("book" . "%a, %t, %u (%y).") ("techreport" . "%a, %t, %i, %u (%y).")
                                     ("proceedings" . "%e, %t in %S, %u (%y).") ("inproceedings" . "%a, %t, %p, in %b, edited by %e, %u (%y)"))
 org-startup-with-inline-images t
 org-ref-clean-bibtex-key-function '(lambda (key) (replace-regexp-in-string ":" "" key))
 org-agenda-before-write-hook '(org-agenda-add-entry-text)
 org-metaup-hook '(org-babel-load-in-session-maybe)
 org-bibtex-headline-format-function #[257 "\300\236A\207" [:title] 3 "\n\n(fn ENTRY)"]
 org-from-is-user-regexp "\\<oub@Utnapischtim\\>\\|\\<Uwe Brauer\\>"
 org-latex-format-drawer-function #[514 "\207" [] 3 "\n\n(fn _ CONTENTS)"]
 org-ref-default-citation-link "citep"
 org-ref-insert-cite-key "C-c c"
 org-babel-pre-tangle-hook '(save-buffer)
 org-tab-first-hook '(org-babel-hide-result-toggle-maybe org-babel-header-arg-expand)
 org-html-table-default-attributes '(:border "2" :cellspacing "0" :cellpadding "6" :frame "border" :rules "all")
 org-babel-load-languages '((emacs-lisp . t) (R . t))
 org-ref-clean-bibtex-entry-hook '(org-ref-bibtex-format-url-if-doi orcb-key-comma org-ref-replace-nonascii orcb-& orcb-%
                                   org-ref-title-case-article orcb-clean-year orcb-key orcb-clean-doi orcb-clean-pages orcb-check-journal
                                   org-ref-sort-bibtex-entry)
 org-load-hook '(my-link-keys org-insert-dblock-bindings)
 org-ref-open-pdf-function 'org-ref-open-pdf-at-point
 org-highlight-latex-and-related '(latex)
 org-ascii-format-drawer-function #[771 "\207" [] 4 "\n\n(fn NAME CONTENTS WIDTH)"]
 org-ref-open-notes-function '(lambda nil (org-show-entry) (outline-show-branches) (outline-show-children) (org-cycle '(64))
                               (recenter-top-bottom 0))
 org-src-lang-modes '(("jupyter-matlab" . matlab) ("jupyter-python" . python) ("ipython" . python) ("C" . c) ("C++" . c++) ("asymptote" . asy)
                      ("bash" . sh) ("beamer" . latex) ("calc" . fundamental) ("cpp" . c++) ("ditaa" . artist) ("dot" . fundamental)
                      ("elisp" . emacs-lisp) ("ocaml" . tuareg) ("screen" . shell-script) ("shell" . sh) ("sqlite" . sql))
 org-src-preserve-indentation t
 org-annotate-file-add-search t
 org-occur-hook '(org-first-headline-recenter)
 org-protocol-protocol-alist '(("capture-html" :protocol "capture-html" :function org-protocol-capture-html--with-pandoc :kill-client t))
 org-export-before-processing-hook '(f-ox-filter-table-column-del)
 org-ref-bibtex-assoc-pdf-with-entry-move-function 'rename-file
 org-agenda-include-diary t
 org-structure-template-alist '(("a" . "export ascii") ("C" . "comment") ("e" . "example") ("E" . "export") ("h" . "export html")
                                ("m" . "src matlab :tangle mieuler.m :exports code  :padline no :eval never-export :wrap latex")
                                ("q" . "quote") ("S" . "src")
                                ("s" . "src matlab :results output latex :exports code  :eval never-export :wrap latex")
                                ("l" . "src latex :results latex replace :exports results :eval t") ("v" . "verse"))
 org-cycle-hook '(org-cycle-hide-archived-subtrees org-cycle-show-empty-lines org-optimize-window-after-visibility-change)
 org-speed-command-hook '(org-speed-command-activate org-babel-speed-command-activate)
 org-capture-mode-hook '(gnorb-org-capture-function)
 org-odt-format-inlinetask-function 'org-odt-format-inlinetask-default-function
 org-html-with-latex 'mathjax
 org-latex-prefer-user-labels t
 org-odt-convert-process "gnumeric"
 org-babel-tangle-lang-exts '(("D" . "d") ("C++" . "cpp") ("perl" . "pl") ("ruby" . "rb") ("python" . "py") ("latex" . "tex")
                              ("emacs-lisp" . "el") ("elisp" . "el"))
 org-return-follows-link t
 org-format-latex-options '(:foreground default :background default :scale 1.5 :html-foreground "Black" :html-background "Transparent"
                            :html-scale 1 :matchers ("begin" "$1" "$" "$$" "\\(" "\\["))
 org-export-with-author nil
 org-latex-listings-langs '((emacs-lisp "Lisp") (lisp "Lisp") (clojure "Lisp") (c "C") (cc "C++") (fortran "fortran") (perl "Perl")
                            (cperl "Perl") (python "Python") (ruby "Ruby") (html "HTML") (xml "XML") (tex "TeX") (latex "[LaTeX]TeX")
                            (shell-script "bash") (gnuplot "Gnuplot") (ocaml "Caml") (caml "Caml") (sql "SQL") (matlab "Matlab")
                            (sqlite "sql") (makefile "make"))
 org-confirm-shell-link-function 'yes-or-no-p
 org-link-parameters '(("eww" :follow eww :store org-eww-store-link) ("rmail" :follow org-rmail-open :store org-rmail-store-link)
                       ("mhe" :follow org-mhe-open :store org-mhe-store-link)
                       ("irc" :follow org-irc-visit :store org-irc-store-link :export org-irc-export)
                       ("info" :follow org-info-open :export org-info-export :store org-info-store-link)
                       ("gnus" :follow org-gnus-open :store org-gnus-store-link)
                       ("docview" :follow org-docview-open :export org-docview-export :store org-docview-store-link)
                       ("bbdb" :follow org-bbdb-open :export org-bbdb-export :complete org-bbdb-complete-link :store org-bbdb-store-link)
                       ("w3m" :store org-w3m-store-link) ("tel")
                       ("printindex" :follow org-ref-index :export
                        #[(path desc format) "\b\301=\205	\300\302!\207" [format latex "printindex"] 2])
                       ("index" :follow #[(path) "\301\b!\207" [path occur] 2] :export
                        #[(path desc format) "\b\302=\205\n\300\303	\"\207" [format path latex "\\index{%s}"] 3])
                       ("bibentry" :follow (lambda (_) (funcall org-ref-cite-onclick-function nil)) :export org-ref-format-bibentry :complete
                        org-bibentry-complete-link :help-echo
                        (lambda (window object position)
                         (when org-ref-show-citation-on-enter
                          (save-excursion (goto-char position)
                           (let ((s (org-ref-format-entry (org-ref-get-bibtex-key-under-cursor))))
                            (with-temp-buffer (insert s) (fill-paragraph) (buffer-string)))
                           )
                          )
                         )
                        :face org-ref-cite-link-face-fn :display full :keymap
                        (keymap (S-up . org-ref-sort-citation-link) (S-right lambda nil (interactive) (org-ref-swap-citation-link 1))
                         (S-left lambda nil (interactive) (org-ref-swap-citation-link -1)) (C-right . org-ref-next-key)
                         (C-left . org-ref-previous-key)
                         (16777337 lambda nil "Paste key at point. Assumes the first thing in the killring is a key." (interactive)
                          (org-ref-insert-key-at-point (car kill-ring)))
                         (16777303 lambda nil "Copy all the keys at point." (interactive)
                          (kill-new (org-element-property :path (org-element-context))))
                         (16777335 lambda nil (interactive) (kill-new (car (org-ref-get-bibtex-key-and-file))))
                         (16777318 lambda nil (interactive)
                          (save-excursion (org-ref-open-citation-at-point) (kill-new (org-ref-format-bibtex-entry-at-point))))
                         (16777319 . org-ref-google-scholar-at-point)
                         (16777317 lambda nil "Email entry at point" (interactive) (org-ref-open-citation-at-point)
                          (org-ref-email-bibtex-entry))
                         (16777315 . org-ref-wos-citing-at-point) (16777330 . org-ref-wos-related-at-point)
                         (16777326 . org-ref-open-notes-at-point) (16777328 . org-ref-open-pdf-at-point)
                         (16777333 . org-ref-open-url-at-point) (16777314 . org-ref-open-citation-at-point) (follow-link . mouse-face)
                         (mouse-3 . org-find-file-at-mouse) (mouse-2 . org-open-at-mouse))
                        )
                       ("Autocites" :follow (lambda (_) (funcall org-ref-cite-onclick-function nil)) :export org-ref-format-Autocites
                        :complete org-Autocites-complete-link :help-echo
                        (lambda (window object position)
                         (when org-ref-show-citation-on-enter
                          (save-excursion (goto-char position)
                           (let ((s (org-ref-format-entry (org-ref-get-bibtex-key-under-cursor))))
                            (with-temp-buffer (insert s) (fill-paragraph) (buffer-string)))
                           )
                          )
                         )
                        :face org-ref-cite-link-face-fn :display full :keymap
                        (keymap (S-up . org-ref-sort-citation-link) (S-right lambda nil (interactive) (org-ref-swap-citation-link 1))
                         (S-left lambda nil (interactive) (org-ref-swap-citation-link -1)) (C-right . org-ref-next-key)
                         (C-left . org-ref-previous-key)
                         (16777337 lambda nil "Paste key at point. Assumes the first thing in the killring is a key." (interactive)
                          (org-ref-insert-key-at-point (car kill-ring)))
                         (16777303 lambda nil "Copy all the keys at point." (interactive)
                          (kill-new (org-element-property :path (org-element-context))))
                         (16777335 lambda nil (interactive) (kill-new (car (org-ref-get-bibtex-key-and-file))))
                         (16777318 lambda nil (interactive)
                          (save-excursion (org-ref-open-citation-at-point) (kill-new (org-ref-format-bibtex-entry-at-point))))
                         (16777319 . org-ref-google-scholar-at-point)
                         (16777317 lambda nil "Email entry at point" (interactive) (org-ref-open-citation-at-point)
                          (org-ref-email-bibtex-entry))
                         (16777315 . org-ref-wos-citing-at-point) (16777330 . org-ref-wos-related-at-point)
                         (16777326 . org-ref-open-notes-at-point) (16777328 . org-ref-open-pdf-at-point)
                         (16777333 . org-ref-open-url-at-point) (16777314 . org-ref-open-citation-at-point) (follow-link . mouse-face)
                         (mouse-3 . org-find-file-at-mouse) (mouse-2 . org-open-at-mouse))
                        )
                       ("autocites" :follow (lambda (_) (funcall org-ref-cite-onclick-function nil)) :export org-ref-format-autocites
                        :complete org-autocites-complete-link :help-echo
                        (lambda (window object position)
                         (when org-ref-show-citation-on-enter
                          (save-excursion (goto-char position)
                           (let ((s (org-ref-format-entry (org-ref-get-bibtex-key-under-cursor))))
                            (with-temp-buffer (insert s) (fill-paragraph) (buffer-string)))
                           )
                          )
                         )
                        :face org-ref-cite-link-face-fn :display full :keymap
                        (keymap (S-up . org-ref-sort-citation-link) (S-right lambda nil (interactive) (org-ref-swap-citation-link 1))
                         (S-left lambda nil (interactive) (org-ref-swap-citation-link -1)) (C-right . org-ref-next-key)
                         (C-left . org-ref-previous-key)
                         (16777337 lambda nil "Paste key at point. Assumes the first thing in the killring is a key." (interactive)
                          (org-ref-insert-key-at-point (car kill-ring)))
                         (16777303 lambda nil "Copy all the keys at point." (interactive)
                          (kill-new (org-element-property :path (org-element-context))))
                         (16777335 lambda nil (interactive) (kill-new (car (org-ref-get-bibtex-key-and-file))))
                         (16777318 lambda nil (interactive)
                          (save-excursion (org-ref-open-citation-at-point) (kill-new (org-ref-format-bibtex-entry-at-point))))
                         (16777319 . org-ref-google-scholar-at-point)
                         (16777317 lambda nil "Email entry at point" (interactive) (org-ref-open-citation-at-point)
                          (org-ref-email-bibtex-entry))
                         (16777315 . org-ref-wos-citing-at-point) (16777330 . org-ref-wos-related-at-point)
                         (16777326 . org-ref-open-notes-at-point) (16777328 . org-ref-open-pdf-at-point)
                         (16777333 . org-ref-open-url-at-point) (16777314 . org-ref-open-citation-at-point) (follow-link . mouse-face)
                         (mouse-3 . org-find-file-at-mouse) (mouse-2 . org-open-at-mouse))
                        )
                       ("supercites" :follow (lambda (_) (funcall org-ref-cite-onclick-function nil)) :export org-ref-format-supercites
                        :complete org-supercites-complete-link :help-echo
                        (lambda (window object position)
                         (when org-ref-show-citation-on-enter
                          (save-excursion (goto-char position)
                           (let ((s (org-ref-format-entry (org-ref-get-bibtex-key-under-cursor))))
                            (with-temp-buffer (insert s) (fill-paragraph) (buffer-string)))
                           )
                          )
                         )
                        :face org-ref-cite-link-face-fn :display full :keymap
                        (keymap (S-up . org-ref-sort-citation-link) (S-right lambda nil (interactive) (org-ref-swap-citation-link 1))
                         (S-left lambda nil (interactive) (org-ref-swap-citation-link -1)) (C-right . org-ref-next-key)
                         (C-left . org-ref-previous-key)
                         (16777337 lambda nil "Paste key at point. Assumes the first thing in the killring is a key." (interactive)
                          (org-ref-insert-key-at-point (car kill-ring)))
                         (16777303 lambda nil "Copy all the keys at point." (interactive)
                          (kill-new (org-element-property :path (org-element-context))))
                         (16777335 lambda nil (interactive) (kill-new (car (org-ref-get-bibtex-key-and-file))))
                         (16777318 lambda nil (interactive)
                          (save-excursion (org-ref-open-citation-at-point) (kill-new (org-ref-format-bibtex-entry-at-point))))
                         (16777319 . org-ref-google-scholar-at-point)
                         (16777317 lambda nil "Email entry at point" (interactive) (org-ref-open-citation-at-point)
                          (org-ref-email-bibtex-entry))
                         (16777315 . org-ref-wos-citing-at-point) (16777330 . org-ref-wos-related-at-point)
                         (16777326 . org-ref-open-notes-at-point) (16777328 . org-ref-open-pdf-at-point)
                         (16777333 . org-ref-open-url-at-point) (16777314 . org-ref-open-citation-at-point) (follow-link . mouse-face)
                         (mouse-3 . org-find-file-at-mouse) (mouse-2 . org-open-at-mouse))
                        )
                       ("Textcites" :follow (lambda (_) (funcall org-ref-cite-onclick-function nil)) :export org-ref-format-Textcites
                        :complete org-Textcites-complete-link :help-echo
                        (lambda (window object position)
                         (when org-ref-show-citation-on-enter
                          (save-excursion (goto-char position)
                           (let ((s (org-ref-format-entry (org-ref-get-bibtex-key-under-cursor))))
                            (with-temp-buffer (insert s) (fill-paragraph) (buffer-string)))
                           )
                          )
                         )
                        :face org-ref-cite-link-face-fn :display full :keymap
                        (keymap (S-up . org-ref-sort-citation-link) (S-right lambda nil (interactive) (org-ref-swap-citation-link 1))
                         (S-left lambda nil (interactive) (org-ref-swap-citation-link -1)) (C-right . org-ref-next-key)
                         (C-left . org-ref-previous-key)
                         (16777337 lambda nil "Paste key at point. Assumes the first thing in the killring is a key." (interactive)
                          (org-ref-insert-key-at-point (car kill-ring)))
                         (16777303 lambda nil "Copy all the keys at point." (interactive)
                          (kill-new (org-element-property :path (org-element-context))))
                         (16777335 lambda nil (interactive) (kill-new (car (org-ref-get-bibtex-key-and-file))))
                         (16777318 lambda nil (interactive)
                          (save-excursion (org-ref-open-citation-at-point) (kill-new (org-ref-format-bibtex-entry-at-point))))
                         (16777319 . org-ref-google-scholar-at-point)
                         (16777317 lambda nil "Email entry at point" (interactive) (org-ref-open-citation-at-point)
                          (org-ref-email-bibtex-entry))
                         (16777315 . org-ref-wos-citing-at-point) (16777330 . org-ref-wos-related-at-point)
                         (16777326 . org-ref-open-notes-at-point) (16777328 . org-ref-open-pdf-at-point)
                         (16777333 . org-ref-open-url-at-point) (16777314 . org-ref-open-citation-at-point) (follow-link . mouse-face)
                         (mouse-3 . org-find-file-at-mouse) (mouse-2 . org-open-at-mouse))
                        )
                       ("textcites" :follow (lambda (_) (funcall org-ref-cite-onclick-function nil)) :export org-ref-format-textcites
                        :complete org-textcites-complete-link :help-echo
                        (lambda (window object position)
                         (when org-ref-show-citation-on-enter
                          (save-excursion (goto-char position)
                           (let ((s (org-ref-format-entry (org-ref-get-bibtex-key-under-cursor))))
                            (with-temp-buffer (insert s) (fill-paragraph) (buffer-string)))
                           )
                          )
                         )
                        :face org-ref-cite-link-face-fn :display full :keymap
                        (keymap (S-up . org-ref-sort-citation-link) (S-right lambda nil (interactive) (org-ref-swap-citation-link 1))
                         (S-left lambda nil (interactive) (org-ref-swap-citation-link -1)) (C-right . org-ref-next-key)
                         (C-left . org-ref-previous-key)
                         (16777337 lambda nil "Paste key at point. Assumes the first thing in the killring is a key." (interactive)
                          (org-ref-insert-key-at-point (car kill-ring)))
                         (16777303 lambda nil "Copy all the keys at point." (interactive)
                          (kill-new (org-element-property :path (org-element-context))))
                         (16777335 lambda nil (interactive) (kill-new (car (org-ref-get-bibtex-key-and-file))))
                         (16777318 lambda nil (interactive)
                          (save-excursion (org-ref-open-citation-at-point) (kill-new (org-ref-format-bibtex-entry-at-point))))
                         (16777319 . org-ref-google-scholar-at-point)
                         (16777317 lambda nil "Email entry at point" (interactive) (org-ref-open-citation-at-point)
                          (org-ref-email-bibtex-entry))
                         (16777315 . org-ref-wos-citing-at-point) (16777330 . org-ref-wos-related-at-point)
                         (16777326 . org-ref-open-notes-at-point) (16777328 . org-ref-open-pdf-at-point)
                         (16777333 . org-ref-open-url-at-point) (16777314 . org-ref-open-citation-at-point) (follow-link . mouse-face)
                         (mouse-3 . org-find-file-at-mouse) (mouse-2 . org-open-at-mouse))
                        )
                       ("Smartcites" :follow (lambda (_) (funcall org-ref-cite-onclick-function nil)) :export org-ref-format-Smartcites
                        :complete org-Smartcites-complete-link :help-echo
                        (lambda (window object position)
                         (when org-ref-show-citation-on-enter
                          (save-excursion (goto-char position)
                           (let ((s (org-ref-format-entry (org-ref-get-bibtex-key-under-cursor))))
                            (with-temp-buffer (insert s) (fill-paragraph) (buffer-string)))
                           )
                          )
                         )
                        :face org-ref-cite-link-face-fn :display full :keymap
                        (keymap (S-up . org-ref-sort-citation-link) (S-right lambda nil (interactive) (org-ref-swap-citation-link 1))
                         (S-left lambda nil (interactive) (org-ref-swap-citation-link -1)) (C-right . org-ref-next-key)
                         (C-left . org-ref-previous-key)
                         (16777337 lambda nil "Paste key at point. Assumes the first thing in the killring is a key." (interactive)
                          (org-ref-insert-key-at-point (car kill-ring)))
                         (16777303 lambda nil "Copy all the keys at point." (interactive)
                          (kill-new (org-element-property :path (org-element-context))))
                         (16777335 lambda nil (interactive) (kill-new (car (org-ref-get-bibtex-key-and-file))))
                         (16777318 lambda nil (interactive)
                          (save-excursion (org-ref-open-citation-at-point) (kill-new (org-ref-format-bibtex-entry-at-point))))
                         (16777319 . org-ref-google-scholar-at-point)
                         (16777317 lambda nil "Email entry at point" (interactive) (org-ref-open-citation-at-point)
                          (org-ref-email-bibtex-entry))
                         (16777315 . org-ref-wos-citing-at-point) (16777330 . org-ref-wos-related-at-point)
                         (16777326 . org-ref-open-notes-at-point) (16777328 . org-ref-open-pdf-at-point)
                         (16777333 . org-ref-open-url-at-point) (16777314 . org-ref-open-citation-at-point) (follow-link . mouse-face)
                         (mouse-3 . org-find-file-at-mouse) (mouse-2 . org-open-at-mouse))
                        )
                       ("smartcites" :follow (lambda (_) (funcall org-ref-cite-onclick-function nil)) :export org-ref-format-smartcites
                        :complete org-smartcites-complete-link :help-echo
                        (lambda (window object position)
                         (when org-ref-show-citation-on-enter
                          (save-excursion (goto-char position)
                           (let ((s (org-ref-format-entry (org-ref-get-bibtex-key-under-cursor))))
                            (with-temp-buffer (insert s) (fill-paragraph) (buffer-string)))
                           )
                          )
                         )
                        :face org-ref-cite-link-face-fn :display full :keymap
                        (keymap (S-up . org-ref-sort-citation-link) (S-right lambda nil (interactive) (org-ref-swap-citation-link 1))
                         (S-left lambda nil (interactive) (org-ref-swap-citation-link -1)) (C-right . org-ref-next-key)
                         (C-left . org-ref-previous-key)
                         (16777337 lambda nil "Paste key at point. Assumes the first thing in the killring is a key." (interactive)
                          (org-ref-insert-key-at-point (car kill-ring)))
                         (16777303 lambda nil "Copy all the keys at point." (interactive)
                          (kill-new (org-element-property :path (org-element-context))))
                         (16777335 lambda nil (interactive) (kill-new (car (org-ref-get-bibtex-key-and-file))))
                         (16777318 lambda nil (interactive)
                          (save-excursion (org-ref-open-citation-at-point) (kill-new (org-ref-format-bibtex-entry-at-point))))
                         (16777319 . org-ref-google-scholar-at-point)
                         (16777317 lambda nil "Email entry at point" (interactive) (org-ref-open-citation-at-point)
                          (org-ref-email-bibtex-entry))
                         (16777315 . org-ref-wos-citing-at-point) (16777330 . org-ref-wos-related-at-point)
                         (16777326 . org-ref-open-notes-at-point) (16777328 . org-ref-open-pdf-at-point)
                         (16777333 . org-ref-open-url-at-point) (16777314 . org-ref-open-citation-at-point) (follow-link . mouse-face)
                         (mouse-3 . org-find-file-at-mouse) (mouse-2 . org-open-at-mouse))
                        )
                       ("footcitetexts" :follow (lambda (_) (funcall org-ref-cite-onclick-function nil)) :export org-ref-format-footcitetexts
                        :complete org-footcitetexts-complete-link :help-echo
                        (lambda (window object position)
                         (when org-ref-show-citation-on-enter
                          (save-excursion (goto-char position)
                           (let ((s (org-ref-format-entry (org-ref-get-bibtex-key-under-cursor))))
                            (with-temp-buffer (insert s) (fill-paragraph) (buffer-string)))
                           )
                          )
                         )
                        :face org-ref-cite-link-face-fn :display full :keymap
                        (keymap (S-up . org-ref-sort-citation-link) (S-right lambda nil (interactive) (org-ref-swap-citation-link 1))
                         (S-left lambda nil (interactive) (org-ref-swap-citation-link -1)) (C-right . org-ref-next-key)
                         (C-left . org-ref-previous-key)
                         (16777337 lambda nil "Paste key at point. Assumes the first thing in the killring is a key." (interactive)
                          (org-ref-insert-key-at-point (car kill-ring)))
                         (16777303 lambda nil "Copy all the keys at point." (interactive)
                          (kill-new (org-element-property :path (org-element-context))))
                         (16777335 lambda nil (interactive) (kill-new (car (org-ref-get-bibtex-key-and-file))))
                         (16777318 lambda nil (interactive)
                          (save-excursion (org-ref-open-citation-at-point) (kill-new (org-ref-format-bibtex-entry-at-point))))
                         (16777319 . org-ref-google-scholar-at-point)
                         (16777317 lambda nil "Email entry at point" (interactive) (org-ref-open-citation-at-point)
                          (org-ref-email-bibtex-entry))
                         (16777315 . org-ref-wos-citing-at-point) (16777330 . org-ref-wos-related-at-point)
                         (16777326 . org-ref-open-notes-at-point) (16777328 . org-ref-open-pdf-at-point)
                         (16777333 . org-ref-open-url-at-point) (16777314 . org-ref-open-citation-at-point) (follow-link . mouse-face)
                         (mouse-3 . org-find-file-at-mouse) (mouse-2 . org-open-at-mouse))
                        )
                       ("footcites" :follow (lambda (_) (funcall org-ref-cite-onclick-function nil)) :export org-ref-format-footcites
                        :complete org-footcites-complete-link :help-echo
                        (lambda (window object position)
                         (when org-ref-show-citation-on-enter
                          (save-excursion (goto-char position)
                           (let ((s (org-ref-format-entry (org-ref-get-bibtex-key-under-cursor))))
                            (with-temp-buffer (insert s) (fill-paragraph) (buffer-string)))
                           )
                          )
                         )
                        :face org-ref-cite-link-face-fn :display full :keymap
                        (keymap (S-up . org-ref-sort-citation-link) (S-right lambda nil (interactive) (org-ref-swap-citation-link 1))
                         (S-left lambda nil (interactive) (org-ref-swap-citation-link -1)) (C-right . org-ref-next-key)
                         (C-left . org-ref-previous-key)
                         (16777337 lambda nil "Paste key at point. Assumes the first thing in the killring is a key." (interactive)
                          (org-ref-insert-key-at-point (car kill-ring)))
                         (16777303 lambda nil "Copy all the keys at point." (interactive)
                          (kill-new (org-element-property :path (org-element-context))))
                         (16777335 lambda nil (interactive) (kill-new (car (org-ref-get-bibtex-key-and-file))))
                         (16777318 lambda nil (interactive)
                          (save-excursion (org-ref-open-citation-at-point) (kill-new (org-ref-format-bibtex-entry-at-point))))
                         (16777319 . org-ref-google-scholar-at-point)
                         (16777317 lambda nil "Email entry at point" (interactive) (org-ref-open-citation-at-point)
                          (org-ref-email-bibtex-entry))
                         (16777315 . org-ref-wos-citing-at-point) (16777330 . org-ref-wos-related-at-point)
                         (16777326 . org-ref-open-notes-at-point) (16777328 . org-ref-open-pdf-at-point)
                         (16777333 . org-ref-open-url-at-point) (16777314 . org-ref-open-citation-at-point) (follow-link . mouse-face)
                         (mouse-3 . org-find-file-at-mouse) (mouse-2 . org-open-at-mouse))
                        )
                       ("Parencites" :follow (lambda (_) (funcall org-ref-cite-onclick-function nil)) :export org-ref-format-Parencites
                        :complete org-Parencites-complete-link :help-echo
                        (lambda (window object position)
                         (when org-ref-show-citation-on-enter
                          (save-excursion (goto-char position)
                           (let ((s (org-ref-format-entry (org-ref-get-bibtex-key-under-cursor))))
                            (with-temp-buffer (insert s) (fill-paragraph) (buffer-string)))
                           )
                          )
                         )
                        :face org-ref-cite-link-face-fn :display full :keymap
                        (keymap (S-up . org-ref-sort-citation-link) (S-right lambda nil (interactive) (org-ref-swap-citation-link 1))
                         (S-left lambda nil (interactive) (org-ref-swap-citation-link -1)) (C-right . org-ref-next-key)
                         (C-left . org-ref-previous-key)
                         (16777337 lambda nil "Paste key at point. Assumes the first thing in the killring is a key." (interactive)
                          (org-ref-insert-key-at-point (car kill-ring)))
                         (16777303 lambda nil "Copy all the keys at point." (interactive)
                          (kill-new (org-element-property :path (org-element-context))))
                         (16777335 lambda nil (interactive) (kill-new (car (org-ref-get-bibtex-key-and-file))))
                         (16777318 lambda nil (interactive)
                          (save-excursion (org-ref-open-citation-at-point) (kill-new (org-ref-format-bibtex-entry-at-point))))
                         (16777319 . org-ref-google-scholar-at-point)
                         (16777317 lambda nil "Email entry at point" (interactive) (org-ref-open-citation-at-point)
                          (org-ref-email-bibtex-entry))
                         (16777315 . org-ref-wos-citing-at-point) (16777330 . org-ref-wos-related-at-point)
                         (16777326 . org-ref-open-notes-at-point) (16777328 . org-ref-open-pdf-at-point)
                         (16777333 . org-ref-open-url-at-point) (16777314 . org-ref-open-citation-at-point) (follow-link . mouse-face)
                         (mouse-3 . org-find-file-at-mouse) (mouse-2 . org-open-at-mouse))
                        )
                       ("parencites" :follow (lambda (_) (funcall org-ref-cite-onclick-function nil)) :export org-ref-format-parencites
                        :complete org-parencites-complete-link :help-echo
                        (lambda (window object position)
                         (when org-ref-show-citation-on-enter
                          (save-excursion (goto-char position)
                           (let ((s (org-ref-format-entry (org-ref-get-bibtex-key-under-cursor))))
                            (with-temp-buffer (insert s) (fill-paragraph) (buffer-string)))
                           )
                          )
                         )
                        :face org-ref-cite-link-face-fn :display full :keymap
                        (keymap (S-up . org-ref-sort-citation-link) (S-right lambda nil (interactive) (org-ref-swap-citation-link 1))
                         (S-left lambda nil (interactive) (org-ref-swap-citation-link -1)) (C-right . org-ref-next-key)
                         (C-left . org-ref-previous-key)
                         (16777337 lambda nil "Paste key at point. Assumes the first thing in the killring is a key." (interactive)
                          (org-ref-insert-key-at-point (car kill-ring)))
                         (16777303 lambda nil "Copy all the keys at point." (interactive)
                          (kill-new (org-element-property :path (org-element-context))))
                         (16777335 lambda nil (interactive) (kill-new (car (org-ref-get-bibtex-key-and-file))))
                         (16777318 lambda nil (interactive)
                          (save-excursion (org-ref-open-citation-at-point) (kill-new (org-ref-format-bibtex-entry-at-point))))
                         (16777319 . org-ref-google-scholar-at-point)
                         (16777317 lambda nil "Email entry at point" (interactive) (org-ref-open-citation-at-point)
                          (org-ref-email-bibtex-entry))
                         (16777315 . org-ref-wos-citing-at-point) (16777330 . org-ref-wos-related-at-point)
                         (16777326 . org-ref-open-notes-at-point) (16777328 . org-ref-open-pdf-at-point)
                         (16777333 . org-ref-open-url-at-point) (16777314 . org-ref-open-citation-at-point) (follow-link . mouse-face)
                         (mouse-3 . org-find-file-at-mouse) (mouse-2 . org-open-at-mouse))
                        )
                       ("Cites" :follow (lambda (_) (funcall org-ref-cite-onclick-function nil)) :export org-ref-format-Cites :complete
                        org-Cites-complete-link :help-echo
                        (lambda (window object position)
                         (when org-ref-show-citation-on-enter
                          (save-excursion (goto-char position)
                           (let ((s (org-ref-format-entry (org-ref-get-bibtex-key-under-cursor))))
                            (with-temp-buffer (insert s) (fill-paragraph) (buffer-string)))
                           )
                          )
                         )
                        :face org-ref-cite-link-face-fn :display full :keymap
                        (keymap (S-up . org-ref-sort-citation-link) (S-right lambda nil (interactive) (org-ref-swap-citation-link 1))
                         (S-left lambda nil (interactive) (org-ref-swap-citation-link -1)) (C-right . org-ref-next-key)
                         (C-left . org-ref-previous-key)
                         (16777337 lambda nil "Paste key at point. Assumes the first thing in the killring is a key." (interactive)
                          (org-ref-insert-key-at-point (car kill-ring)))
                         (16777303 lambda nil "Copy all the keys at point." (interactive)
                          (kill-new (org-element-property :path (org-element-context))))
                         (16777335 lambda nil (interactive) (kill-new (car (org-ref-get-bibtex-key-and-file))))
                         (16777318 lambda nil (interactive)
                          (save-excursion (org-ref-open-citation-at-point) (kill-new (org-ref-format-bibtex-entry-at-point))))
                         (16777319 . org-ref-google-scholar-at-point)
                         (16777317 lambda nil "Email entry at point" (interactive) (org-ref-open-citation-at-point)
                          (org-ref-email-bibtex-entry))
                         (16777315 . org-ref-wos-citing-at-point) (16777330 . org-ref-wos-related-at-point)
                         (16777326 . org-ref-open-notes-at-point) (16777328 . org-ref-open-pdf-at-point)
                         (16777333 . org-ref-open-url-at-point) (16777314 . org-ref-open-citation-at-point) (follow-link . mouse-face)
                         (mouse-3 . org-find-file-at-mouse) (mouse-2 . org-open-at-mouse))
                        )
                       ("cites" :follow (lambda (_) (funcall org-ref-cite-onclick-function nil)) :export org-ref-format-cites :complete
                        org-cites-complete-link :help-echo
                        (lambda (window object position)
                         (when org-ref-show-citation-on-enter
                          (save-excursion (goto-char position)
                           (let ((s (org-ref-format-entry (org-ref-get-bibtex-key-under-cursor))))
                            (with-temp-buffer (insert s) (fill-paragraph) (buffer-string)))
                           )
                          )
                         )
                        :face org-ref-cite-link-face-fn :display full :keymap
                        (keymap (S-up . org-ref-sort-citation-link) (S-right lambda nil (interactive) (org-ref-swap-citation-link 1))
                         (S-left lambda nil (interactive) (org-ref-swap-citation-link -1)) (C-right . org-ref-next-key)
                         (C-left . org-ref-previous-key)
                         (16777337 lambda nil "Paste key at point. Assumes the first thing in the killring is a key." (interactive)
                          (org-ref-insert-key-at-point (car kill-ring)))
                         (16777303 lambda nil "Copy all the keys at point." (interactive)
                          (kill-new (org-element-property :path (org-element-context))))
                         (16777335 lambda nil (interactive) (kill-new (car (org-ref-get-bibtex-key-and-file))))
                         (16777318 lambda nil (interactive)
                          (save-excursion (org-ref-open-citation-at-point) (kill-new (org-ref-format-bibtex-entry-at-point))))
                         (16777319 . org-ref-google-scholar-at-point)
                         (16777317 lambda nil "Email entry at point" (interactive) (org-ref-open-citation-at-point)
                          (org-ref-email-bibtex-entry))
                         (16777315 . org-ref-wos-citing-at-point) (16777330 . org-ref-wos-related-at-point)
                         (16777326 . org-ref-open-notes-at-point) (16777328 . org-ref-open-pdf-at-point)
                         (16777333 . org-ref-open-url-at-point) (16777314 . org-ref-open-citation-at-point) (follow-link . mouse-face)
                         (mouse-3 . org-find-file-at-mouse) (mouse-2 . org-open-at-mouse))
                        )
                       ("fnotecite" :follow (lambda (_) (funcall org-ref-cite-onclick-function nil)) :export org-ref-format-fnotecite
                        :complete org-fnotecite-complete-link :help-echo
                        (lambda (window object position)
                         (when org-ref-show-citation-on-enter
                          (save-excursion (goto-char position)
                           (let ((s (org-ref-format-entry (org-ref-get-bibtex-key-under-cursor))))
                            (with-temp-buffer (insert s) (fill-paragraph) (buffer-string)))
                           )
                          )
                         )
                        :face org-ref-cite-link-face-fn :display full :keymap
                        (keymap (S-up . org-ref-sort-citation-link) (S-right lambda nil (interactive) (org-ref-swap-citation-link 1))
                         (S-left lambda nil (interactive) (org-ref-swap-citation-link -1)) (C-right . org-ref-next-key)
                         (C-left . org-ref-previous-key)
                         (16777337 lambda nil "Paste key at point. Assumes the first thing in the killring is a key." (interactive)
                          (org-ref-insert-key-at-point (car kill-ring)))
                         (16777303 lambda nil "Copy all the keys at point." (interactive)
                          (kill-new (org-element-property :path (org-element-context))))
                         (16777335 lambda nil (interactive) (kill-new (car (org-ref-get-bibtex-key-and-file))))
                         (16777318 lambda nil (interactive)
                          (save-excursion (org-ref-open-citation-at-point) (kill-new (org-ref-format-bibtex-entry-at-point))))
                         (16777319 . org-ref-google-scholar-at-point)
                         (16777317 lambda nil "Email entry at point" (interactive) (org-ref-open-citation-at-point)
                          (org-ref-email-bibtex-entry))
                         (16777315 . org-ref-wos-citing-at-point) (16777330 . org-ref-wos-related-at-point)
                         (16777326 . org-ref-open-notes-at-point) (16777328 . org-ref-open-pdf-at-point)
                         (16777333 . org-ref-open-url-at-point) (16777314 . org-ref-open-citation-at-point) (follow-link . mouse-face)
                         (mouse-3 . org-find-file-at-mouse) (mouse-2 . org-open-at-mouse))
                        )
                       ("Pnotecite" :follow (lambda (_) (funcall org-ref-cite-onclick-function nil)) :export org-ref-format-Pnotecite
                        :complete org-Pnotecite-complete-link :help-echo
                        (lambda (window object position)
                         (when org-ref-show-citation-on-enter
                          (save-excursion (goto-char position)
                           (let ((s (org-ref-format-entry (org-ref-get-bibtex-key-under-cursor))))
                            (with-temp-buffer (insert s) (fill-paragraph) (buffer-string)))
                           )
                          )
                         )
                        :face org-ref-cite-link-face-fn :display full :keymap
                        (keymap (S-up . org-ref-sort-citation-link) (S-right lambda nil (interactive) (org-ref-swap-citation-link 1))
                         (S-left lambda nil (interactive) (org-ref-swap-citation-link -1)) (C-right . org-ref-next-key)
                         (C-left . org-ref-previous-key)
                         (16777337 lambda nil "Paste key at point. Assumes the first thing in the killring is a key." (interactive)
                          (org-ref-insert-key-at-point (car kill-ring)))
                         (16777303 lambda nil "Copy all the keys at point." (interactive)
                          (kill-new (org-element-property :path (org-element-context))))
                         (16777335 lambda nil (interactive) (kill-new (car (org-ref-get-bibtex-key-and-file))))
                         (16777318 lambda nil (interactive)
                          (save-excursion (org-ref-open-citation-at-point) (kill-new (org-ref-format-bibtex-entry-at-point))))
                         (16777319 . org-ref-google-scholar-at-point)
                         (16777317 lambda nil "Email entry at point" (interactive) (org-ref-open-citation-at-point)
                          (org-ref-email-bibtex-entry))
                         (16777315 . org-ref-wos-citing-at-point) (16777330 . org-ref-wos-related-at-point)
                         (16777326 . org-ref-open-notes-at-point) (16777328 . org-ref-open-pdf-at-point)
                         (16777333 . org-ref-open-url-at-point) (16777314 . org-ref-open-citation-at-point) (follow-link . mouse-face)
                         (mouse-3 . org-find-file-at-mouse) (mouse-2 . org-open-at-mouse))
                        )
                       ("pnotecite" :follow (lambda (_) (funcall org-ref-cite-onclick-function nil)) :export org-ref-format-pnotecite
                        :complete org-pnotecite-complete-link :help-echo
                        (lambda (window object position)
                         (when org-ref-show-citation-on-enter
                          (save-excursion (goto-char position)
                           (let ((s (org-ref-format-entry (org-ref-get-bibtex-key-under-cursor))))
                            (with-temp-buffer (insert s) (fill-paragraph) (buffer-string)))
                           )
                          )
                         )
                        :face org-ref-cite-link-face-fn :display full :keymap
                        (keymap (S-up . org-ref-sort-citation-link) (S-right lambda nil (interactive) (org-ref-swap-citation-link 1))
                         (S-left lambda nil (interactive) (org-ref-swap-citation-link -1)) (C-right . org-ref-next-key)
                         (C-left . org-ref-previous-key)
                         (16777337 lambda nil "Paste key at point. Assumes the first thing in the killring is a key." (interactive)
                          (org-ref-insert-key-at-point (car kill-ring)))
                         (16777303 lambda nil "Copy all the keys at point." (interactive)
                          (kill-new (org-element-property :path (org-element-context))))
                         (16777335 lambda nil (interactive) (kill-new (car (org-ref-get-bibtex-key-and-file))))
                         (16777318 lambda nil (interactive)
                          (save-excursion (org-ref-open-citation-at-point) (kill-new (org-ref-format-bibtex-entry-at-point))))
                         (16777319 . org-ref-google-scholar-at-point)
                         (16777317 lambda nil "Email entry at point" (interactive) (org-ref-open-citation-at-point)
                          (org-ref-email-bibtex-entry))
                         (16777315 . org-ref-wos-citing-at-point) (16777330 . org-ref-wos-related-at-point)
                         (16777326 . org-ref-open-notes-at-point) (16777328 . org-ref-open-pdf-at-point)
                         (16777333 . org-ref-open-url-at-point) (16777314 . org-ref-open-citation-at-point) (follow-link . mouse-face)
                         (mouse-3 . org-find-file-at-mouse) (mouse-2 . org-open-at-mouse))
                        )
                       ("Notecite" :follow (lambda (_) (funcall org-ref-cite-onclick-function nil)) :export org-ref-format-Notecite :complete
                        org-Notecite-complete-link :help-echo
                        (lambda (window object position)
                         (when org-ref-show-citation-on-enter
                          (save-excursion (goto-char position)
                           (let ((s (org-ref-format-entry (org-ref-get-bibtex-key-under-cursor))))
                            (with-temp-buffer (insert s) (fill-paragraph) (buffer-string)))
                           )
                          )
                         )
                        :face org-ref-cite-link-face-fn :display full :keymap
                        (keymap (S-up . org-ref-sort-citation-link) (S-right lambda nil (interactive) (org-ref-swap-citation-link 1))
                         (S-left lambda nil (interactive) (org-ref-swap-citation-link -1)) (C-right . org-ref-next-key)
                         (C-left . org-ref-previous-key)
                         (16777337 lambda nil "Paste key at point. Assumes the first thing in the killring is a key." (interactive)
                          (org-ref-insert-key-at-point (car kill-ring)))
                         (16777303 lambda nil "Copy all the keys at point." (interactive)
                          (kill-new (org-element-property :path (org-element-context))))
                         (16777335 lambda nil (interactive) (kill-new (car (org-ref-get-bibtex-key-and-file))))
                         (16777318 lambda nil (interactive)
                          (save-excursion (org-ref-open-citation-at-point) (kill-new (org-ref-format-bibtex-entry-at-point))))
                         (16777319 . org-ref-google-scholar-at-point)
                         (16777317 lambda nil "Email entry at point" (interactive) (org-ref-open-citation-at-point)
                          (org-ref-email-bibtex-entry))
                         (16777315 . org-ref-wos-citing-at-point) (16777330 . org-ref-wos-related-at-point)
                         (16777326 . org-ref-open-notes-at-point) (16777328 . org-ref-open-pdf-at-point)
                         (16777333 . org-ref-open-url-at-point) (16777314 . org-ref-open-citation-at-point) (follow-link . mouse-face)
                         (mouse-3 . org-find-file-at-mouse) (mouse-2 . org-open-at-mouse))
                        )
                       ("notecite" :follow (lambda (_) (funcall org-ref-cite-onclick-function nil)) :export org-ref-format-notecite :complete
                        org-notecite-complete-link :help-echo
                        (lambda (window object position)
                         (when org-ref-show-citation-on-enter
                          (save-excursion (goto-char position)
                           (let ((s (org-ref-format-entry (org-ref-get-bibtex-key-under-cursor))))
                            (with-temp-buffer (insert s) (fill-paragraph) (buffer-string)))
                           )
                          )
                         )
                        :face org-ref-cite-link-face-fn :display full :keymap
                        (keymap (S-up . org-ref-sort-citation-link) (S-right lambda nil (interactive) (org-ref-swap-citation-link 1))
                         (S-left lambda nil (interactive) (org-ref-swap-citation-link -1)) (C-right . org-ref-next-key)
                         (C-left . org-ref-previous-key)
                         (16777337 lambda nil "Paste key at point. Assumes the first thing in the killring is a key." (interactive)
                          (org-ref-insert-key-at-point (car kill-ring)))
                         (16777303 lambda nil "Copy all the keys at point." (interactive)
                          (kill-new (org-element-property :path (org-element-context))))
                         (16777335 lambda nil (interactive) (kill-new (car (org-ref-get-bibtex-key-and-file))))
                         (16777318 lambda nil (interactive)
                          (save-excursion (org-ref-open-citation-at-point) (kill-new (org-ref-format-bibtex-entry-at-point))))
                         (16777319 . org-ref-google-scholar-at-point)
                         (16777317 lambda nil "Email entry at point" (interactive) (org-ref-open-citation-at-point)
                          (org-ref-email-bibtex-entry))
                         (16777315 . org-ref-wos-citing-at-point) (16777330 . org-ref-wos-related-at-point)
                         (16777326 . org-ref-open-notes-at-point) (16777328 . org-ref-open-pdf-at-point)
                         (16777333 . org-ref-open-url-at-point) (16777314 . org-ref-open-citation-at-point) (follow-link . mouse-face)
                         (mouse-3 . org-find-file-at-mouse) (mouse-2 . org-open-at-mouse))
                        )
                       ("footfullcite" :follow (lambda (_) (funcall org-ref-cite-onclick-function nil)) :export org-ref-format-footfullcite
                        :complete org-footfullcite-complete-link :help-echo
                        (lambda (window object position)
                         (when org-ref-show-citation-on-enter
                          (save-excursion (goto-char position)
                           (let ((s (org-ref-format-entry (org-ref-get-bibtex-key-under-cursor))))
                            (with-temp-buffer (insert s) (fill-paragraph) (buffer-string)))
                           )
                          )
                         )
                        :face org-ref-cite-link-face-fn :display full :keymap
                        (keymap (S-up . org-ref-sort-citation-link) (S-right lambda nil (interactive) (org-ref-swap-citation-link 1))
                         (S-left lambda nil (interactive) (org-ref-swap-citation-link -1)) (C-right . org-ref-next-key)
                         (C-left . org-ref-previous-key)
                         (16777337 lambda nil "Paste key at point. Assumes the first thing in the killring is a key." (interactive)
                          (org-ref-insert-key-at-point (car kill-ring)))
                         (16777303 lambda nil "Copy all the keys at point." (interactive)
                          (kill-new (org-element-property :path (org-element-context))))
                         (16777335 lambda nil (interactive) (kill-new (car (org-ref-get-bibtex-key-and-file))))
                         (16777318 lambda nil (interactive)
                          (save-excursion (org-ref-open-citation-at-point) (kill-new (org-ref-format-bibtex-entry-at-point))))
                         (16777319 . org-ref-google-scholar-at-point)
                         (16777317 lambda nil "Email entry at point" (interactive) (org-ref-open-citation-at-point)
                          (org-ref-email-bibtex-entry))
                         (16777315 . org-ref-wos-citing-at-point) (16777330 . org-ref-wos-related-at-point)
                         (16777326 . org-ref-open-notes-at-point) (16777328 . org-ref-open-pdf-at-point)
                         (16777333 . org-ref-open-url-at-point) (16777314 . org-ref-open-citation-at-point) (follow-link . mouse-face)
                         (mouse-3 . org-find-file-at-mouse) (mouse-2 . org-open-at-mouse))
                        )
                       ("fullcite" :follow (lambda (_) (funcall org-ref-cite-onclick-function nil)) :export org-ref-format-fullcite :complete
                        org-fullcite-complete-link :help-echo
                        (lambda (window object position)
                         (when org-ref-show-citation-on-enter
                          (save-excursion (goto-char position)
                           (let ((s (org-ref-format-entry (org-ref-get-bibtex-key-under-cursor))))
                            (with-temp-buffer (insert s) (fill-paragraph) (buffer-string)))
                           )
                          )
                         )
                        :face org-ref-cite-link-face-fn :display full :keymap
                        (keymap (S-up . org-ref-sort-citation-link) (S-right lambda nil (interactive) (org-ref-swap-citation-link 1))
                         (S-left lambda nil (interactive) (org-ref-swap-citation-link -1)) (C-right . org-ref-next-key)
                         (C-left . org-ref-previous-key)
                         (16777337 lambda nil "Paste key at point. Assumes the first thing in the killring is a key." (interactive)
                          (org-ref-insert-key-at-point (car kill-ring)))
                         (16777303 lambda nil "Copy all the keys at point." (interactive)
                          (kill-new (org-element-property :path (org-element-context))))
                         (16777335 lambda nil (interactive) (kill-new (car (org-ref-get-bibtex-key-and-file))))
                         (16777318 lambda nil (interactive)
                          (save-excursion (org-ref-open-citation-at-point) (kill-new (org-ref-format-bibtex-entry-at-point))))
                         (16777319 . org-ref-google-scholar-at-point)
                         (16777317 lambda nil "Email entry at point" (interactive) (org-ref-open-citation-at-point)
                          (org-ref-email-bibtex-entry))
                         (16777315 . org-ref-wos-citing-at-point) (16777330 . org-ref-wos-related-at-point)
                         (16777326 . org-ref-open-notes-at-point) (16777328 . org-ref-open-pdf-at-point)
                         (16777333 . org-ref-open-url-at-point) (16777314 . org-ref-open-citation-at-point) (follow-link . mouse-face)
                         (mouse-3 . org-find-file-at-mouse) (mouse-2 . org-open-at-mouse))
                        )
                       ("citeurl" :follow (lambda (_) (funcall org-ref-cite-onclick-function nil)) :export org-ref-format-citeurl :complete
                        org-citeurl-complete-link :help-echo
                        (lambda (window object position)
                         (when org-ref-show-citation-on-enter
                          (save-excursion (goto-char position)
                           (let ((s (org-ref-format-entry (org-ref-get-bibtex-key-under-cursor))))
                            (with-temp-buffer (insert s) (fill-paragraph) (buffer-string)))
                           )
                          )
                         )
                        :face org-ref-cite-link-face-fn :display full :keymap
                        (keymap (S-up . org-ref-sort-citation-link) (S-right lambda nil (interactive) (org-ref-swap-citation-link 1))
                         (S-left lambda nil (interactive) (org-ref-swap-citation-link -1)) (C-right . org-ref-next-key)
                         (C-left . org-ref-previous-key)
                         (16777337 lambda nil "Paste key at point. Assumes the first thing in the killring is a key." (interactive)
                          (org-ref-insert-key-at-point (car kill-ring)))
                         (16777303 lambda nil "Copy all the keys at point." (interactive)
                          (kill-new (org-element-property :path (org-element-context))))
                         (16777335 lambda nil (interactive) (kill-new (car (org-ref-get-bibtex-key-and-file))))
                         (16777318 lambda nil (interactive)
                          (save-excursion (org-ref-open-citation-at-point) (kill-new (org-ref-format-bibtex-entry-at-point))))
                         (16777319 . org-ref-google-scholar-at-point)
                         (16777317 lambda nil "Email entry at point" (interactive) (org-ref-open-citation-at-point)
                          (org-ref-email-bibtex-entry))
                         (16777315 . org-ref-wos-citing-at-point) (16777330 . org-ref-wos-related-at-point)
                         (16777326 . org-ref-open-notes-at-point) (16777328 . org-ref-open-pdf-at-point)
                         (16777333 . org-ref-open-url-at-point) (16777314 . org-ref-open-citation-at-point) (follow-link . mouse-face)
                         (mouse-3 . org-find-file-at-mouse) (mouse-2 . org-open-at-mouse))
                        )
                       ("citedate*" :follow (lambda (_) (funcall org-ref-cite-onclick-function nil)) :export org-ref-format-citedate*
                        :complete org-citedate*-complete-link :help-echo
                        (lambda (window object position)
                         (when org-ref-show-citation-on-enter
                          (save-excursion (goto-char position)
                           (let ((s (org-ref-format-entry (org-ref-get-bibtex-key-under-cursor))))
                            (with-temp-buffer (insert s) (fill-paragraph) (buffer-string)))
                           )
                          )
                         )
                        :face org-ref-cite-link-face-fn :display full :keymap
                        (keymap (S-up . org-ref-sort-citation-link) (S-right lambda nil (interactive) (org-ref-swap-citation-link 1))
                         (S-left lambda nil (interactive) (org-ref-swap-citation-link -1)) (C-right . org-ref-next-key)
                         (C-left . org-ref-previous-key)
                         (16777337 lambda nil "Paste key at point. Assumes the first thing in the killring is a key." (interactive)
                          (org-ref-insert-key-at-point (car kill-ring)))
                         (16777303 lambda nil "Copy all the keys at point." (interactive)
                          (kill-new (org-element-property :path (org-element-context))))
                         (16777335 lambda nil (interactive) (kill-new (car (org-ref-get-bibtex-key-and-file))))
                         (16777318 lambda nil (interactive)
                          (save-excursion (org-ref-open-citation-at-point) (kill-new (org-ref-format-bibtex-entry-at-point))))
                         (16777319 . org-ref-google-scholar-at-point)
                         (16777317 lambda nil "Email entry at point" (interactive) (org-ref-open-citation-at-point)
                          (org-ref-email-bibtex-entry))
                         (16777315 . org-ref-wos-citing-at-point) (16777330 . org-ref-wos-related-at-point)
                         (16777326 . org-ref-open-notes-at-point) (16777328 . org-ref-open-pdf-at-point)
                         (16777333 . org-ref-open-url-at-point) (16777314 . org-ref-open-citation-at-point) (follow-link . mouse-face)
                         (mouse-3 . org-find-file-at-mouse) (mouse-2 . org-open-at-mouse))
                        )
                       ("citedate" :follow (lambda (_) (funcall org-ref-cite-onclick-function nil)) :export org-ref-format-citedate :complete
                        org-citedate-complete-link :help-echo
                        (lambda (window object position)
                         (when org-ref-show-citation-on-enter
                          (save-excursion (goto-char position)
                           (let ((s (org-ref-format-entry (org-ref-get-bibtex-key-under-cursor))))
                            (with-temp-buffer (insert s) (fill-paragraph) (buffer-string)))
                           )
                          )
                         )
                        :face org-ref-cite-link-face-fn :display full :keymap
                        (keymap (S-up . org-ref-sort-citation-link) (S-right lambda nil (interactive) (org-ref-swap-citation-link 1))
                         (S-left lambda nil (interactive) (org-ref-swap-citation-link -1)) (C-right . org-ref-next-key)
                         (C-left . org-ref-previous-key)
                         (16777337 lambda nil "Paste key at point. Assumes the first thing in the killring is a key." (interactive)
                          (org-ref-insert-key-at-point (car kill-ring)))
                         (16777303 lambda nil "Copy all the keys at point." (interactive)
                          (kill-new (org-element-property :path (org-element-context))))
                         (16777335 lambda nil (interactive) (kill-new (car (org-ref-get-bibtex-key-and-file))))
                         (16777318 lambda nil (interactive)
                          (save-excursion (org-ref-open-citation-at-point) (kill-new (org-ref-format-bibtex-entry-at-point))))
                         (16777319 . org-ref-google-scholar-at-point)
                         (16777317 lambda nil "Email entry at point" (interactive) (org-ref-open-citation-at-point)
                          (org-ref-email-bibtex-entry))
                         (16777315 . org-ref-wos-citing-at-point) (16777330 . org-ref-wos-related-at-point)
                         (16777326 . org-ref-open-notes-at-point) (16777328 . org-ref-open-pdf-at-point)
                         (16777333 . org-ref-open-url-at-point) (16777314 . org-ref-open-citation-at-point) (follow-link . mouse-face)
                         (mouse-3 . org-find-file-at-mouse) (mouse-2 . org-open-at-mouse))
                        )
                       ("citetitle*" :follow (lambda (_) (funcall org-ref-cite-onclick-function nil)) :export org-ref-format-citetitle*
                        :complete org-citetitle*-complete-link :help-echo
                        (lambda (window object position)
                         (when org-ref-show-citation-on-enter
                          (save-excursion (goto-char position)
                           (let ((s (org-ref-format-entry (org-ref-get-bibtex-key-under-cursor))))
                            (with-temp-buffer (insert s) (fill-paragraph) (buffer-string)))
                           )
                          )
                         )
                        :face org-ref-cite-link-face-fn :display full :keymap
                        (keymap (S-up . org-ref-sort-citation-link) (S-right lambda nil (interactive) (org-ref-swap-citation-link 1))
                         (S-left lambda nil (interactive) (org-ref-swap-citation-link -1)) (C-right . org-ref-next-key)
                         (C-left . org-ref-previous-key)
                         (16777337 lambda nil "Paste key at point. Assumes the first thing in the killring is a key." (interactive)
                          (org-ref-insert-key-at-point (car kill-ring)))
                         (16777303 lambda nil "Copy all the keys at point." (interactive)
                          (kill-new (org-element-property :path (org-element-context))))
                         (16777335 lambda nil (interactive) (kill-new (car (org-ref-get-bibtex-key-and-file))))
                         (16777318 lambda nil (interactive)
                          (save-excursion (org-ref-open-citation-at-point) (kill-new (org-ref-format-bibtex-entry-at-point))))
                         (16777319 . org-ref-google-scholar-at-point)
                         (16777317 lambda nil "Email entry at point" (interactive) (org-ref-open-citation-at-point)
                          (org-ref-email-bibtex-entry))
                         (16777315 . org-ref-wos-citing-at-point) (16777330 . org-ref-wos-related-at-point)
                         (16777326 . org-ref-open-notes-at-point) (16777328 . org-ref-open-pdf-at-point)
                         (16777333 . org-ref-open-url-at-point) (16777314 . org-ref-open-citation-at-point) (follow-link . mouse-face)
                         (mouse-3 . org-find-file-at-mouse) (mouse-2 . org-open-at-mouse))
                        )
                       ("citetitle" :follow (lambda (_) (funcall org-ref-cite-onclick-function nil)) :export org-ref-format-citetitle
                        :complete org-citetitle-complete-link :help-echo
                        (lambda (window object position)
                         (when org-ref-show-citation-on-enter
                          (save-excursion (goto-char position)
                           (let ((s (org-ref-format-entry (org-ref-get-bibtex-key-under-cursor))))
                            (with-temp-buffer (insert s) (fill-paragraph) (buffer-string)))
                           )
                          )
                         )
                        :face org-ref-cite-link-face-fn :display full :keymap
                        (keymap (S-up . org-ref-sort-citation-link) (S-right lambda nil (interactive) (org-ref-swap-citation-link 1))
                         (S-left lambda nil (interactive) (org-ref-swap-citation-link -1)) (C-right . org-ref-next-key)
                         (C-left . org-ref-previous-key)
                         (16777337 lambda nil "Paste key at point. Assumes the first thing in the killring is a key." (interactive)
                          (org-ref-insert-key-at-point (car kill-ring)))
                         (16777303 lambda nil "Copy all the keys at point." (interactive)
                          (kill-new (org-element-property :path (org-element-context))))
                         (16777335 lambda nil (interactive) (kill-new (car (org-ref-get-bibtex-key-and-file))))
                         (16777318 lambda nil (interactive)
                          (save-excursion (org-ref-open-citation-at-point) (kill-new (org-ref-format-bibtex-entry-at-point))))
                         (16777319 . org-ref-google-scholar-at-point)
                         (16777317 lambda nil "Email entry at point" (interactive) (org-ref-open-citation-at-point)
                          (org-ref-email-bibtex-entry))
                         (16777315 . org-ref-wos-citing-at-point) (16777330 . org-ref-wos-related-at-point)
                         (16777326 . org-ref-open-notes-at-point) (16777328 . org-ref-open-pdf-at-point)
                         (16777333 . org-ref-open-url-at-point) (16777314 . org-ref-open-citation-at-point) (follow-link . mouse-face)
                         (mouse-3 . org-find-file-at-mouse) (mouse-2 . org-open-at-mouse))
                        )
                       ("Citeauthor*" :follow (lambda (_) (funcall org-ref-cite-onclick-function nil)) :export org-ref-format-Citeauthor*
                        :complete org-Citeauthor*-complete-link :help-echo
                        (lambda (window object position)
                         (when org-ref-show-citation-on-enter
                          (save-excursion (goto-char position)
                           (let ((s (org-ref-format-entry (org-ref-get-bibtex-key-under-cursor))))
                            (with-temp-buffer (insert s) (fill-paragraph) (buffer-string)))
                           )
                          )
                         )
                        :face org-ref-cite-link-face-fn :display full :keymap
                        (keymap (S-up . org-ref-sort-citation-link) (S-right lambda nil (interactive) (org-ref-swap-citation-link 1))
                         (S-left lambda nil (interactive) (org-ref-swap-citation-link -1)) (C-right . org-ref-next-key)
                         (C-left . org-ref-previous-key)
                         (16777337 lambda nil "Paste key at point. Assumes the first thing in the killring is a key." (interactive)
                          (org-ref-insert-key-at-point (car kill-ring)))
                         (16777303 lambda nil "Copy all the keys at point." (interactive)
                          (kill-new (org-element-property :path (org-element-context))))
                         (16777335 lambda nil (interactive) (kill-new (car (org-ref-get-bibtex-key-and-file))))
                         (16777318 lambda nil (interactive)
                          (save-excursion (org-ref-open-citation-at-point) (kill-new (org-ref-format-bibtex-entry-at-point))))
                         (16777319 . org-ref-google-scholar-at-point)
                         (16777317 lambda nil "Email entry at point" (interactive) (org-ref-open-citation-at-point)
                          (org-ref-email-bibtex-entry))
                         (16777315 . org-ref-wos-citing-at-point) (16777330 . org-ref-wos-related-at-point)
                         (16777326 . org-ref-open-notes-at-point) (16777328 . org-ref-open-pdf-at-point)
                         (16777333 . org-ref-open-url-at-point) (16777314 . org-ref-open-citation-at-point) (follow-link . mouse-face)
                         (mouse-3 . org-find-file-at-mouse) (mouse-2 . org-open-at-mouse))
                        )
                       ("Autocite*" :follow (lambda (_) (funcall org-ref-cite-onclick-function nil)) :export org-ref-format-Autocite*
                        :complete org-Autocite*-complete-link :help-echo
                        (lambda (window object position)
                         (when org-ref-show-citation-on-enter
                          (save-excursion (goto-char position)
                           (let ((s (org-ref-format-entry (org-ref-get-bibtex-key-under-cursor))))
                            (with-temp-buffer (insert s) (fill-paragraph) (buffer-string)))
                           )
                          )
                         )
                        :face org-ref-cite-link-face-fn :display full :keymap
                        (keymap (S-up . org-ref-sort-citation-link) (S-right lambda nil (interactive) (org-ref-swap-citation-link 1))
                         (S-left lambda nil (interactive) (org-ref-swap-citation-link -1)) (C-right . org-ref-next-key)
                         (C-left . org-ref-previous-key)
                         (16777337 lambda nil "Paste key at point. Assumes the first thing in the killring is a key." (interactive)
                          (org-ref-insert-key-at-point (car kill-ring)))
                         (16777303 lambda nil "Copy all the keys at point." (interactive)
                          (kill-new (org-element-property :path (org-element-context))))
                         (16777335 lambda nil (interactive) (kill-new (car (org-ref-get-bibtex-key-and-file))))
                         (16777318 lambda nil (interactive)
                          (save-excursion (org-ref-open-citation-at-point) (kill-new (org-ref-format-bibtex-entry-at-point))))
                         (16777319 . org-ref-google-scholar-at-point)
                         (16777317 lambda nil "Email entry at point" (interactive) (org-ref-open-citation-at-point)
                          (org-ref-email-bibtex-entry))
                         (16777315 . org-ref-wos-citing-at-point) (16777330 . org-ref-wos-related-at-point)
                         (16777326 . org-ref-open-notes-at-point) (16777328 . org-ref-open-pdf-at-point)
                         (16777333 . org-ref-open-url-at-point) (16777314 . org-ref-open-citation-at-point) (follow-link . mouse-face)
                         (mouse-3 . org-find-file-at-mouse) (mouse-2 . org-open-at-mouse))
                        )
                       ("autocite*" :follow (lambda (_) (funcall org-ref-cite-onclick-function nil)) :export org-ref-format-autocite*
                        :complete org-autocite*-complete-link :help-echo
                        (lambda (window object position)
                         (when org-ref-show-citation-on-enter
                          (save-excursion (goto-char position)
                           (let ((s (org-ref-format-entry (org-ref-get-bibtex-key-under-cursor))))
                            (with-temp-buffer (insert s) (fill-paragraph) (buffer-string)))
                           )
                          )
                         )
                        :face org-ref-cite-link-face-fn :display full :keymap
                        (keymap (S-up . org-ref-sort-citation-link) (S-right lambda nil (interactive) (org-ref-swap-citation-link 1))
                         (S-left lambda nil (interactive) (org-ref-swap-citation-link -1)) (C-right . org-ref-next-key)
                         (C-left . org-ref-previous-key)
                         (16777337 lambda nil "Paste key at point. Assumes the first thing in the killring is a key." (interactive)
                          (org-ref-insert-key-at-point (car kill-ring)))
                         (16777303 lambda nil "Copy all the keys at point." (interactive)
                          (kill-new (org-element-property :path (org-element-context))))
                         (16777335 lambda nil (interactive) (kill-new (car (org-ref-get-bibtex-key-and-file))))
                         (16777318 lambda nil (interactive)
                          (save-excursion (org-ref-open-citation-at-point) (kill-new (org-ref-format-bibtex-entry-at-point))))
                         (16777319 . org-ref-google-scholar-at-point)
                         (16777317 lambda nil "Email entry at point" (interactive) (org-ref-open-citation-at-point)
                          (org-ref-email-bibtex-entry))
                         (16777315 . org-ref-wos-citing-at-point) (16777330 . org-ref-wos-related-at-point)
                         (16777326 . org-ref-open-notes-at-point) (16777328 . org-ref-open-pdf-at-point)
                         (16777333 . org-ref-open-url-at-point) (16777314 . org-ref-open-citation-at-point) (follow-link . mouse-face)
                         (mouse-3 . org-find-file-at-mouse) (mouse-2 . org-open-at-mouse))
                        )
                       ("Autocite" :follow (lambda (_) (funcall org-ref-cite-onclick-function nil)) :export org-ref-format-Autocite :complete
                        org-Autocite-complete-link :help-echo
                        (lambda (window object position)
                         (when org-ref-show-citation-on-enter
                          (save-excursion (goto-char position)
                           (let ((s (org-ref-format-entry (org-ref-get-bibtex-key-under-cursor))))
                            (with-temp-buffer (insert s) (fill-paragraph) (buffer-string)))
                           )
                          )
                         )
                        :face org-ref-cite-link-face-fn :display full :keymap
                        (keymap (S-up . org-ref-sort-citation-link) (S-right lambda nil (interactive) (org-ref-swap-citation-link 1))
                         (S-left lambda nil (interactive) (org-ref-swap-citation-link -1)) (C-right . org-ref-next-key)
                         (C-left . org-ref-previous-key)
                         (16777337 lambda nil "Paste key at point. Assumes the first thing in the killring is a key." (interactive)
                          (org-ref-insert-key-at-point (car kill-ring)))
                         (16777303 lambda nil "Copy all the keys at point." (interactive)
                          (kill-new (org-element-property :path (org-element-context))))
                         (16777335 lambda nil (interactive) (kill-new (car (org-ref-get-bibtex-key-and-file))))
                         (16777318 lambda nil (interactive)
                          (save-excursion (org-ref-open-citation-at-point) (kill-new (org-ref-format-bibtex-entry-at-point))))
                         (16777319 . org-ref-google-scholar-at-point)
                         (16777317 lambda nil "Email entry at point" (interactive) (org-ref-open-citation-at-point)
                          (org-ref-email-bibtex-entry))
                         (16777315 . org-ref-wos-citing-at-point) (16777330 . org-ref-wos-related-at-point)
                         (16777326 . org-ref-open-notes-at-point) (16777328 . org-ref-open-pdf-at-point)
                         (16777333 . org-ref-open-url-at-point) (16777314 . org-ref-open-citation-at-point) (follow-link . mouse-face)
                         (mouse-3 . org-find-file-at-mouse) (mouse-2 . org-open-at-mouse))
                        )
                       ("autocite" :follow (lambda (_) (funcall org-ref-cite-onclick-function nil)) :export org-ref-format-autocite :complete
                        org-autocite-complete-link :help-echo
                        (lambda (window object position)
                         (when org-ref-show-citation-on-enter
                          (save-excursion (goto-char position)
                           (let ((s (org-ref-format-entry (org-ref-get-bibtex-key-under-cursor))))
                            (with-temp-buffer (insert s) (fill-paragraph) (buffer-string)))
                           )
                          )
                         )
                        :face org-ref-cite-link-face-fn :display full :keymap
                        (keymap (S-up . org-ref-sort-citation-link) (S-right lambda nil (interactive) (org-ref-swap-citation-link 1))
                         (S-left lambda nil (interactive) (org-ref-swap-citation-link -1)) (C-right . org-ref-next-key)
                         (C-left . org-ref-previous-key)
                         (16777337 lambda nil "Paste key at point. Assumes the first thing in the killring is a key." (interactive)
                          (org-ref-insert-key-at-point (car kill-ring)))
                         (16777303 lambda nil "Copy all the keys at point." (interactive)
                          (kill-new (org-element-property :path (org-element-context))))
                         (16777335 lambda nil (interactive) (kill-new (car (org-ref-get-bibtex-key-and-file))))
                         (16777318 lambda nil (interactive)
                          (save-excursion (org-ref-open-citation-at-point) (kill-new (org-ref-format-bibtex-entry-at-point))))
                         (16777319 . org-ref-google-scholar-at-point)
                         (16777317 lambda nil "Email entry at point" (interactive) (org-ref-open-citation-at-point)
                          (org-ref-email-bibtex-entry))
                         (16777315 . org-ref-wos-citing-at-point) (16777330 . org-ref-wos-related-at-point)
                         (16777326 . org-ref-open-notes-at-point) (16777328 . org-ref-open-pdf-at-point)
                         (16777333 . org-ref-open-url-at-point) (16777314 . org-ref-open-citation-at-point) (follow-link . mouse-face)
                         (mouse-3 . org-find-file-at-mouse) (mouse-2 . org-open-at-mouse))
                        )
                       ("supercite" :follow (lambda (_) (funcall org-ref-cite-onclick-function nil)) :export org-ref-format-supercite
                        :complete org-supercite-complete-link :help-echo
                        (lambda (window object position)
                         (when org-ref-show-citation-on-enter
                          (save-excursion (goto-char position)
                           (let ((s (org-ref-format-entry (org-ref-get-bibtex-key-under-cursor))))
                            (with-temp-buffer (insert s) (fill-paragraph) (buffer-string)))
                           )
                          )
                         )
                        :face org-ref-cite-link-face-fn :display full :keymap
                        (keymap (S-up . org-ref-sort-citation-link) (S-right lambda nil (interactive) (org-ref-swap-citation-link 1))
                         (S-left lambda nil (interactive) (org-ref-swap-citation-link -1)) (C-right . org-ref-next-key)
                         (C-left . org-ref-previous-key)
                         (16777337 lambda nil "Paste key at point. Assumes the first thing in the killring is a key." (interactive)
                          (org-ref-insert-key-at-point (car kill-ring)))
                         (16777303 lambda nil "Copy all the keys at point." (interactive)
                          (kill-new (org-element-property :path (org-element-context))))
                         (16777335 lambda nil (interactive) (kill-new (car (org-ref-get-bibtex-key-and-file))))
                         (16777318 lambda nil (interactive)
                          (save-excursion (org-ref-open-citation-at-point) (kill-new (org-ref-format-bibtex-entry-at-point))))
                         (16777319 . org-ref-google-scholar-at-point)
                         (16777317 lambda nil "Email entry at point" (interactive) (org-ref-open-citation-at-point)
                          (org-ref-email-bibtex-entry))
                         (16777315 . org-ref-wos-citing-at-point) (16777330 . org-ref-wos-related-at-point)
                         (16777326 . org-ref-open-notes-at-point) (16777328 . org-ref-open-pdf-at-point)
                         (16777333 . org-ref-open-url-at-point) (16777314 . org-ref-open-citation-at-point) (follow-link . mouse-face)
                         (mouse-3 . org-find-file-at-mouse) (mouse-2 . org-open-at-mouse))
                        )
                       ("parencite*" :follow (lambda (_) (funcall org-ref-cite-onclick-function nil)) :export org-ref-format-parencite*
                        :complete org-parencite*-complete-link :help-echo
                        (lambda (window object position)
                         (when org-ref-show-citation-on-enter
                          (save-excursion (goto-char position)
                           (let ((s (org-ref-format-entry (org-ref-get-bibtex-key-under-cursor))))
                            (with-temp-buffer (insert s) (fill-paragraph) (buffer-string)))
                           )
                          )
                         )
                        :face org-ref-cite-link-face-fn :display full :keymap
                        (keymap (S-up . org-ref-sort-citation-link) (S-right lambda nil (interactive) (org-ref-swap-citation-link 1))
                         (S-left lambda nil (interactive) (org-ref-swap-citation-link -1)) (C-right . org-ref-next-key)
                         (C-left . org-ref-previous-key)
                         (16777337 lambda nil "Paste key at point. Assumes the first thing in the killring is a key." (interactive)
                          (org-ref-insert-key-at-point (car kill-ring)))
                         (16777303 lambda nil "Copy all the keys at point." (interactive)
                          (kill-new (org-element-property :path (org-element-context))))
                         (16777335 lambda nil (interactive) (kill-new (car (org-ref-get-bibtex-key-and-file))))
                         (16777318 lambda nil (interactive)
                          (save-excursion (org-ref-open-citation-at-point) (kill-new (org-ref-format-bibtex-entry-at-point))))
                         (16777319 . org-ref-google-scholar-at-point)
                         (16777317 lambda nil "Email entry at point" (interactive) (org-ref-open-citation-at-point)
                          (org-ref-email-bibtex-entry))
                         (16777315 . org-ref-wos-citing-at-point) (16777330 . org-ref-wos-related-at-point)
                         (16777326 . org-ref-open-notes-at-point) (16777328 . org-ref-open-pdf-at-point)
                         (16777333 . org-ref-open-url-at-point) (16777314 . org-ref-open-citation-at-point) (follow-link . mouse-face)
                         (mouse-3 . org-find-file-at-mouse) (mouse-2 . org-open-at-mouse))
                        )
                       ("cite*" :follow (lambda (_) (funcall org-ref-cite-onclick-function nil)) :export org-ref-format-cite* :complete
                        org-cite*-complete-link :help-echo
                        (lambda (window object position)
                         (when org-ref-show-citation-on-enter
                          (save-excursion (goto-char position)
                           (let ((s (org-ref-format-entry (org-ref-get-bibtex-key-under-cursor))))
                            (with-temp-buffer (insert s) (fill-paragraph) (buffer-string)))
                           )
                          )
                         )
                        :face org-ref-cite-link-face-fn :display full :keymap
                        (keymap (S-up . org-ref-sort-citation-link) (S-right lambda nil (interactive) (org-ref-swap-citation-link 1))
                         (S-left lambda nil (interactive) (org-ref-swap-citation-link -1)) (C-right . org-ref-next-key)
                         (C-left . org-ref-previous-key)
                         (16777337 lambda nil "Paste key at point. Assumes the first thing in the killring is a key." (interactive)
                          (org-ref-insert-key-at-point (car kill-ring)))
                         (16777303 lambda nil "Copy all the keys at point." (interactive)
                          (kill-new (org-element-property :path (org-element-context))))
                         (16777335 lambda nil (interactive) (kill-new (car (org-ref-get-bibtex-key-and-file))))
                         (16777318 lambda nil (interactive)
                          (save-excursion (org-ref-open-citation-at-point) (kill-new (org-ref-format-bibtex-entry-at-point))))
                         (16777319 . org-ref-google-scholar-at-point)
                         (16777317 lambda nil "Email entry at point" (interactive) (org-ref-open-citation-at-point)
                          (org-ref-email-bibtex-entry))
                         (16777315 . org-ref-wos-citing-at-point) (16777330 . org-ref-wos-related-at-point)
                         (16777326 . org-ref-open-notes-at-point) (16777328 . org-ref-open-pdf-at-point)
                         (16777333 . org-ref-open-url-at-point) (16777314 . org-ref-open-citation-at-point) (follow-link . mouse-face)
                         (mouse-3 . org-find-file-at-mouse) (mouse-2 . org-open-at-mouse))
                        )
                       ("Smartcite" :follow (lambda (_) (funcall org-ref-cite-onclick-function nil)) :export org-ref-format-Smartcite
                        :complete org-Smartcite-complete-link :help-echo
                        (lambda (window object position)
                         (when org-ref-show-citation-on-enter
                          (save-excursion (goto-char position)
                           (let ((s (org-ref-format-entry (org-ref-get-bibtex-key-under-cursor))))
                            (with-temp-buffer (insert s) (fill-paragraph) (buffer-string)))
                           )
                          )
                         )
                        :face org-ref-cite-link-face-fn :display full :keymap
                        (keymap (S-up . org-ref-sort-citation-link) (S-right lambda nil (interactive) (org-ref-swap-citation-link 1))
                         (S-left lambda nil (interactive) (org-ref-swap-citation-link -1)) (C-right . org-ref-next-key)
                         (C-left . org-ref-previous-key)
                         (16777337 lambda nil "Paste key at point. Assumes the first thing in the killring is a key." (interactive)
                          (org-ref-insert-key-at-point (car kill-ring)))
                         (16777303 lambda nil "Copy all the keys at point." (interactive)
                          (kill-new (org-element-property :path (org-element-context))))
                         (16777335 lambda nil (interactive) (kill-new (car (org-ref-get-bibtex-key-and-file))))
                         (16777318 lambda nil (interactive)
                          (save-excursion (org-ref-open-citation-at-point) (kill-new (org-ref-format-bibtex-entry-at-point))))
                         (16777319 . org-ref-google-scholar-at-point)
                         (16777317 lambda nil "Email entry at point" (interactive) (org-ref-open-citation-at-point)
                          (org-ref-email-bibtex-entry))
                         (16777315 . org-ref-wos-citing-at-point) (16777330 . org-ref-wos-related-at-point)
                         (16777326 . org-ref-open-notes-at-point) (16777328 . org-ref-open-pdf-at-point)
                         (16777333 . org-ref-open-url-at-point) (16777314 . org-ref-open-citation-at-point) (follow-link . mouse-face)
                         (mouse-3 . org-find-file-at-mouse) (mouse-2 . org-open-at-mouse))
                        )
                       ("smartcite" :follow (lambda (_) (funcall org-ref-cite-onclick-function nil)) :export org-ref-format-smartcite
                        :complete org-smartcite-complete-link :help-echo
                        (lambda (window object position)
                         (when org-ref-show-citation-on-enter
                          (save-excursion (goto-char position)
                           (let ((s (org-ref-format-entry (org-ref-get-bibtex-key-under-cursor))))
                            (with-temp-buffer (insert s) (fill-paragraph) (buffer-string)))
                           )
                          )
                         )
                        :face org-ref-cite-link-face-fn :display full :keymap
                        (keymap (S-up . org-ref-sort-citation-link) (S-right lambda nil (interactive) (org-ref-swap-citation-link 1))
                         (S-left lambda nil (interactive) (org-ref-swap-citation-link -1)) (C-right . org-ref-next-key)
                         (C-left . org-ref-previous-key)
                         (16777337 lambda nil "Paste key at point. Assumes the first thing in the killring is a key." (interactive)
                          (org-ref-insert-key-at-point (car kill-ring)))
                         (16777303 lambda nil "Copy all the keys at point." (interactive)
                          (kill-new (org-element-property :path (org-element-context))))
                         (16777335 lambda nil (interactive) (kill-new (car (org-ref-get-bibtex-key-and-file))))
                         (16777318 lambda nil (interactive)
                          (save-excursion (org-ref-open-citation-at-point) (kill-new (org-ref-format-bibtex-entry-at-point))))
                         (16777319 . org-ref-google-scholar-at-point)
                         (16777317 lambda nil "Email entry at point" (interactive) (org-ref-open-citation-at-point)
                          (org-ref-email-bibtex-entry))
                         (16777315 . org-ref-wos-citing-at-point) (16777330 . org-ref-wos-related-at-point)
                         (16777326 . org-ref-open-notes-at-point) (16777328 . org-ref-open-pdf-at-point)
                         (16777333 . org-ref-open-url-at-point) (16777314 . org-ref-open-citation-at-point) (follow-link . mouse-face)
                         (mouse-3 . org-find-file-at-mouse) (mouse-2 . org-open-at-mouse))
                        )
                       ("Textcite" :follow (lambda (_) (funcall org-ref-cite-onclick-function nil)) :export org-ref-format-Textcite :complete
                        org-Textcite-complete-link :help-echo
                        (lambda (window object position)
                         (when org-ref-show-citation-on-enter
                          (save-excursion (goto-char position)
                           (let ((s (org-ref-format-entry (org-ref-get-bibtex-key-under-cursor))))
                            (with-temp-buffer (insert s) (fill-paragraph) (buffer-string)))
                           )
                          )
                         )
                        :face org-ref-cite-link-face-fn :display full :keymap
                        (keymap (S-up . org-ref-sort-citation-link) (S-right lambda nil (interactive) (org-ref-swap-citation-link 1))
                         (S-left lambda nil (interactive) (org-ref-swap-citation-link -1)) (C-right . org-ref-next-key)
                         (C-left . org-ref-previous-key)
                         (16777337 lambda nil "Paste key at point. Assumes the first thing in the killring is a key." (interactive)
                          (org-ref-insert-key-at-point (car kill-ring)))
                         (16777303 lambda nil "Copy all the keys at point." (interactive)
                          (kill-new (org-element-property :path (org-element-context))))
                         (16777335 lambda nil (interactive) (kill-new (car (org-ref-get-bibtex-key-and-file))))
                         (16777318 lambda nil (interactive)
                          (save-excursion (org-ref-open-citation-at-point) (kill-new (org-ref-format-bibtex-entry-at-point))))
                         (16777319 . org-ref-google-scholar-at-point)
                         (16777317 lambda nil "Email entry at point" (interactive) (org-ref-open-citation-at-point)
                          (org-ref-email-bibtex-entry))
                         (16777315 . org-ref-wos-citing-at-point) (16777330 . org-ref-wos-related-at-point)
                         (16777326 . org-ref-open-notes-at-point) (16777328 . org-ref-open-pdf-at-point)
                         (16777333 . org-ref-open-url-at-point) (16777314 . org-ref-open-citation-at-point) (follow-link . mouse-face)
                         (mouse-3 . org-find-file-at-mouse) (mouse-2 . org-open-at-mouse))
                        )
                       ("textcite" :follow (lambda (_) (funcall org-ref-cite-onclick-function nil)) :export org-ref-format-textcite :complete
                        org-textcite-complete-link :help-echo
                        (lambda (window object position)
                         (when org-ref-show-citation-on-enter
                          (save-excursion (goto-char position)
                           (let ((s (org-ref-format-entry (org-ref-get-bibtex-key-under-cursor))))
                            (with-temp-buffer (insert s) (fill-paragraph) (buffer-string)))
                           )
                          )
                         )
                        :face org-ref-cite-link-face-fn :display full :keymap
                        (keymap (S-up . org-ref-sort-citation-link) (S-right lambda nil (interactive) (org-ref-swap-citation-link 1))
                         (S-left lambda nil (interactive) (org-ref-swap-citation-link -1)) (C-right . org-ref-next-key)
                         (C-left . org-ref-previous-key)
                         (16777337 lambda nil "Paste key at point. Assumes the first thing in the killring is a key." (interactive)
                          (org-ref-insert-key-at-point (car kill-ring)))
                         (16777303 lambda nil "Copy all the keys at point." (interactive)
                          (kill-new (org-element-property :path (org-element-context))))
                         (16777335 lambda nil (interactive) (kill-new (car (org-ref-get-bibtex-key-and-file))))
                         (16777318 lambda nil (interactive)
                          (save-excursion (org-ref-open-citation-at-point) (kill-new (org-ref-format-bibtex-entry-at-point))))
                         (16777319 . org-ref-google-scholar-at-point)
                         (16777317 lambda nil "Email entry at point" (interactive) (org-ref-open-citation-at-point)
                          (org-ref-email-bibtex-entry))
                         (16777315 . org-ref-wos-citing-at-point) (16777330 . org-ref-wos-related-at-point)
                         (16777326 . org-ref-open-notes-at-point) (16777328 . org-ref-open-pdf-at-point)
                         (16777333 . org-ref-open-url-at-point) (16777314 . org-ref-open-citation-at-point) (follow-link . mouse-face)
                         (mouse-3 . org-find-file-at-mouse) (mouse-2 . org-open-at-mouse))
                        )
                       ("footcitetext" :follow (lambda (_) (funcall org-ref-cite-onclick-function nil)) :export org-ref-format-footcitetext
                        :complete org-footcitetext-complete-link :help-echo
                        (lambda (window object position)
                         (when org-ref-show-citation-on-enter
                          (save-excursion (goto-char position)
                           (let ((s (org-ref-format-entry (org-ref-get-bibtex-key-under-cursor))))
                            (with-temp-buffer (insert s) (fill-paragraph) (buffer-string)))
                           )
                          )
                         )
                        :face org-ref-cite-link-face-fn :display full :keymap
                        (keymap (S-up . org-ref-sort-citation-link) (S-right lambda nil (interactive) (org-ref-swap-citation-link 1))
                         (S-left lambda nil (interactive) (org-ref-swap-citation-link -1)) (C-right . org-ref-next-key)
                         (C-left . org-ref-previous-key)
                         (16777337 lambda nil "Paste key at point. Assumes the first thing in the killring is a key." (interactive)
                          (org-ref-insert-key-at-point (car kill-ring)))
                         (16777303 lambda nil "Copy all the keys at point." (interactive)
                          (kill-new (org-element-property :path (org-element-context))))
                         (16777335 lambda nil (interactive) (kill-new (car (org-ref-get-bibtex-key-and-file))))
                         (16777318 lambda nil (interactive)
                          (save-excursion (org-ref-open-citation-at-point) (kill-new (org-ref-format-bibtex-entry-at-point))))
                         (16777319 . org-ref-google-scholar-at-point)
                         (16777317 lambda nil "Email entry at point" (interactive) (org-ref-open-citation-at-point)
                          (org-ref-email-bibtex-entry))
                         (16777315 . org-ref-wos-citing-at-point) (16777330 . org-ref-wos-related-at-point)
                         (16777326 . org-ref-open-notes-at-point) (16777328 . org-ref-open-pdf-at-point)
                         (16777333 . org-ref-open-url-at-point) (16777314 . org-ref-open-citation-at-point) (follow-link . mouse-face)
                         (mouse-3 . org-find-file-at-mouse) (mouse-2 . org-open-at-mouse))
                        )
                       ("footcite" :follow (lambda (_) (funcall org-ref-cite-onclick-function nil)) :export org-ref-format-footcite :complete
                        org-footcite-complete-link :help-echo
                        (lambda (window object position)
                         (when org-ref-show-citation-on-enter
                          (save-excursion (goto-char position)
                           (let ((s (org-ref-format-entry (org-ref-get-bibtex-key-under-cursor))))
                            (with-temp-buffer (insert s) (fill-paragraph) (buffer-string)))
                           )
                          )
                         )
                        :face org-ref-cite-link-face-fn :display full :keymap
                        (keymap (S-up . org-ref-sort-citation-link) (S-right lambda nil (interactive) (org-ref-swap-citation-link 1))
                         (S-left lambda nil (interactive) (org-ref-swap-citation-link -1)) (C-right . org-ref-next-key)
                         (C-left . org-ref-previous-key)
                         (16777337 lambda nil "Paste key at point. Assumes the first thing in the killring is a key." (interactive)
                          (org-ref-insert-key-at-point (car kill-ring)))
                         (16777303 lambda nil "Copy all the keys at point." (interactive)
                          (kill-new (org-element-property :path (org-element-context))))
                         (16777335 lambda nil (interactive) (kill-new (car (org-ref-get-bibtex-key-and-file))))
                         (16777318 lambda nil (interactive)
                          (save-excursion (org-ref-open-citation-at-point) (kill-new (org-ref-format-bibtex-entry-at-point))))
                         (16777319 . org-ref-google-scholar-at-point)
                         (16777317 lambda nil "Email entry at point" (interactive) (org-ref-open-citation-at-point)
                          (org-ref-email-bibtex-entry))
                         (16777315 . org-ref-wos-citing-at-point) (16777330 . org-ref-wos-related-at-point)
                         (16777326 . org-ref-open-notes-at-point) (16777328 . org-ref-open-pdf-at-point)
                         (16777333 . org-ref-open-url-at-point) (16777314 . org-ref-open-citation-at-point) (follow-link . mouse-face)
                         (mouse-3 . org-find-file-at-mouse) (mouse-2 . org-open-at-mouse))
                        )
                       ("Parencite" :follow (lambda (_) (funcall org-ref-cite-onclick-function nil)) :export org-ref-format-Parencite
                        :complete org-Parencite-complete-link :help-echo
                        (lambda (window object position)
                         (when org-ref-show-citation-on-enter
                          (save-excursion (goto-char position)
                           (let ((s (org-ref-format-entry (org-ref-get-bibtex-key-under-cursor))))
                            (with-temp-buffer (insert s) (fill-paragraph) (buffer-string)))
                           )
                          )
                         )
                        :face org-ref-cite-link-face-fn :display full :keymap
                        (keymap (S-up . org-ref-sort-citation-link) (S-right lambda nil (interactive) (org-ref-swap-citation-link 1))
                         (S-left lambda nil (interactive) (org-ref-swap-citation-link -1)) (C-right . org-ref-next-key)
                         (C-left . org-ref-previous-key)
                         (16777337 lambda nil "Paste key at point. Assumes the first thing in the killring is a key." (interactive)
                          (org-ref-insert-key-at-point (car kill-ring)))
                         (16777303 lambda nil "Copy all the keys at point." (interactive)
                          (kill-new (org-element-property :path (org-element-context))))
                         (16777335 lambda nil (interactive) (kill-new (car (org-ref-get-bibtex-key-and-file))))
                         (16777318 lambda nil (interactive)
                          (save-excursion (org-ref-open-citation-at-point) (kill-new (org-ref-format-bibtex-entry-at-point))))
                         (16777319 . org-ref-google-scholar-at-point)
                         (16777317 lambda nil "Email entry at point" (interactive) (org-ref-open-citation-at-point)
                          (org-ref-email-bibtex-entry))
                         (16777315 . org-ref-wos-citing-at-point) (16777330 . org-ref-wos-related-at-point)
                         (16777326 . org-ref-open-notes-at-point) (16777328 . org-ref-open-pdf-at-point)
                         (16777333 . org-ref-open-url-at-point) (16777314 . org-ref-open-citation-at-point) (follow-link . mouse-face)
                         (mouse-3 . org-find-file-at-mouse) (mouse-2 . org-open-at-mouse))
                        )
                       ("parencite" :follow (lambda (_) (funcall org-ref-cite-onclick-function nil)) :export org-ref-format-parencite
                        :complete org-parencite-complete-link :help-echo
                        (lambda (window object position)
                         (when org-ref-show-citation-on-enter
                          (save-excursion (goto-char position)
                           (let ((s (org-ref-format-entry (org-ref-get-bibtex-key-under-cursor))))
                            (with-temp-buffer (insert s) (fill-paragraph) (buffer-string)))
                           )
                          )
                         )
                        :face org-ref-cite-link-face-fn :display full :keymap
                        (keymap (S-up . org-ref-sort-citation-link) (S-right lambda nil (interactive) (org-ref-swap-citation-link 1))
                         (S-left lambda nil (interactive) (org-ref-swap-citation-link -1)) (C-right . org-ref-next-key)
                         (C-left . org-ref-previous-key)
                         (16777337 lambda nil "Paste key at point. Assumes the first thing in the killring is a key." (interactive)
                          (org-ref-insert-key-at-point (car kill-ring)))
                         (16777303 lambda nil "Copy all the keys at point." (interactive)
                          (kill-new (org-element-property :path (org-element-context))))
                         (16777335 lambda nil (interactive) (kill-new (car (org-ref-get-bibtex-key-and-file))))
                         (16777318 lambda nil (interactive)
                          (save-excursion (org-ref-open-citation-at-point) (kill-new (org-ref-format-bibtex-entry-at-point))))
                         (16777319 . org-ref-google-scholar-at-point)
                         (16777317 lambda nil "Email entry at point" (interactive) (org-ref-open-citation-at-point)
                          (org-ref-email-bibtex-entry))
                         (16777315 . org-ref-wos-citing-at-point) (16777330 . org-ref-wos-related-at-point)
                         (16777326 . org-ref-open-notes-at-point) (16777328 . org-ref-open-pdf-at-point)
                         (16777333 . org-ref-open-url-at-point) (16777314 . org-ref-open-citation-at-point) (follow-link . mouse-face)
                         (mouse-3 . org-find-file-at-mouse) (mouse-2 . org-open-at-mouse))
                        )
                       ("Cite" :follow (lambda (_) (funcall org-ref-cite-onclick-function nil)) :export org-ref-format-Cite :complete
                        org-Cite-complete-link :help-echo
                        (lambda (window object position)
                         (when org-ref-show-citation-on-enter
                          (save-excursion (goto-char position)
                           (let ((s (org-ref-format-entry (org-ref-get-bibtex-key-under-cursor))))
                            (with-temp-buffer (insert s) (fill-paragraph) (buffer-string)))
                           )
                          )
                         )
                        :face org-ref-cite-link-face-fn :display full :keymap
                        (keymap (S-up . org-ref-sort-citation-link) (S-right lambda nil (interactive) (org-ref-swap-citation-link 1))
                         (S-left lambda nil (interactive) (org-ref-swap-citation-link -1)) (C-right . org-ref-next-key)
                         (C-left . org-ref-previous-key)
                         (16777337 lambda nil "Paste key at point. Assumes the first thing in the killring is a key." (interactive)
                          (org-ref-insert-key-at-point (car kill-ring)))
                         (16777303 lambda nil "Copy all the keys at point." (interactive)
                          (kill-new (org-element-property :path (org-element-context))))
                         (16777335 lambda nil (interactive) (kill-new (car (org-ref-get-bibtex-key-and-file))))
                         (16777318 lambda nil (interactive)
                          (save-excursion (org-ref-open-citation-at-point) (kill-new (org-ref-format-bibtex-entry-at-point))))
                         (16777319 . org-ref-google-scholar-at-point)
                         (16777317 lambda nil "Email entry at point" (interactive) (org-ref-open-citation-at-point)
                          (org-ref-email-bibtex-entry))
                         (16777315 . org-ref-wos-citing-at-point) (16777330 . org-ref-wos-related-at-point)
                         (16777326 . org-ref-open-notes-at-point) (16777328 . org-ref-open-pdf-at-point)
                         (16777333 . org-ref-open-url-at-point) (16777314 . org-ref-open-citation-at-point) (follow-link . mouse-face)
                         (mouse-3 . org-find-file-at-mouse) (mouse-2 . org-open-at-mouse))
                        )
                       ("Citeauthor" :follow (lambda (_) (funcall org-ref-cite-onclick-function nil)) :export org-ref-format-Citeauthor
                        :complete org-Citeauthor-complete-link :help-echo
                        (lambda (window object position)
                         (when org-ref-show-citation-on-enter
                          (save-excursion (goto-char position)
                           (let ((s (org-ref-format-entry (org-ref-get-bibtex-key-under-cursor))))
                            (with-temp-buffer (insert s) (fill-paragraph) (buffer-string)))
                           )
                          )
                         )
                        :face org-ref-cite-link-face-fn :display full :keymap
                        (keymap (S-up . org-ref-sort-citation-link) (S-right lambda nil (interactive) (org-ref-swap-citation-link 1))
                         (S-left lambda nil (interactive) (org-ref-swap-citation-link -1)) (C-right . org-ref-next-key)
                         (C-left . org-ref-previous-key)
                         (16777337 lambda nil "Paste key at point. Assumes the first thing in the killring is a key." (interactive)
                          (org-ref-insert-key-at-point (car kill-ring)))
                         (16777303 lambda nil "Copy all the keys at point." (interactive)
                          (kill-new (org-element-property :path (org-element-context))))
                         (16777335 lambda nil (interactive) (kill-new (car (org-ref-get-bibtex-key-and-file))))
                         (16777318 lambda nil (interactive)
                          (save-excursion (org-ref-open-citation-at-point) (kill-new (org-ref-format-bibtex-entry-at-point))))
                         (16777319 . org-ref-google-scholar-at-point)
                         (16777317 lambda nil "Email entry at point" (interactive) (org-ref-open-citation-at-point)
                          (org-ref-email-bibtex-entry))
                         (16777315 . org-ref-wos-citing-at-point) (16777330 . org-ref-wos-related-at-point)
                         (16777326 . org-ref-open-notes-at-point) (16777328 . org-ref-open-pdf-at-point)
                         (16777333 . org-ref-open-url-at-point) (16777314 . org-ref-open-citation-at-point) (follow-link . mouse-face)
                         (mouse-3 . org-find-file-at-mouse) (mouse-2 . org-open-at-mouse))
                        )
                       ("Citealp" :follow (lambda (_) (funcall org-ref-cite-onclick-function nil)) :export org-ref-format-Citealp :complete
                        org-Citealp-complete-link :help-echo
                        (lambda (window object position)
                         (when org-ref-show-citation-on-enter
                          (save-excursion (goto-char position)
                           (let ((s (org-ref-format-entry (org-ref-get-bibtex-key-under-cursor))))
                            (with-temp-buffer (insert s) (fill-paragraph) (buffer-string)))
                           )
                          )
                         )
                        :face org-ref-cite-link-face-fn :display full :keymap
                        (keymap (S-up . org-ref-sort-citation-link) (S-right lambda nil (interactive) (org-ref-swap-citation-link 1))
                         (S-left lambda nil (interactive) (org-ref-swap-citation-link -1)) (C-right . org-ref-next-key)
                         (C-left . org-ref-previous-key)
                         (16777337 lambda nil "Paste key at point. Assumes the first thing in the killring is a key." (interactive)
                          (org-ref-insert-key-at-point (car kill-ring)))
                         (16777303 lambda nil "Copy all the keys at point." (interactive)
                          (kill-new (org-element-property :path (org-element-context))))
                         (16777335 lambda nil (interactive) (kill-new (car (org-ref-get-bibtex-key-and-file))))
                         (16777318 lambda nil (interactive)
                          (save-excursion (org-ref-open-citation-at-point) (kill-new (org-ref-format-bibtex-entry-at-point))))
                         (16777319 . org-ref-google-scholar-at-point)
                         (16777317 lambda nil "Email entry at point" (interactive) (org-ref-open-citation-at-point)
                          (org-ref-email-bibtex-entry))
                         (16777315 . org-ref-wos-citing-at-point) (16777330 . org-ref-wos-related-at-point)
                         (16777326 . org-ref-open-notes-at-point) (16777328 . org-ref-open-pdf-at-point)
                         (16777333 . org-ref-open-url-at-point) (16777314 . org-ref-open-citation-at-point) (follow-link . mouse-face)
                         (mouse-3 . org-find-file-at-mouse) (mouse-2 . org-open-at-mouse))
                        )
                       ("Citealt" :follow (lambda (_) (funcall org-ref-cite-onclick-function nil)) :export org-ref-format-Citealt :complete
                        org-Citealt-complete-link :help-echo
                        (lambda (window object position)
                         (when org-ref-show-citation-on-enter
                          (save-excursion (goto-char position)
                           (let ((s (org-ref-format-entry (org-ref-get-bibtex-key-under-cursor))))
                            (with-temp-buffer (insert s) (fill-paragraph) (buffer-string)))
                           )
                          )
                         )
                        :face org-ref-cite-link-face-fn :display full :keymap
                        (keymap (S-up . org-ref-sort-citation-link) (S-right lambda nil (interactive) (org-ref-swap-citation-link 1))
                         (S-left lambda nil (interactive) (org-ref-swap-citation-link -1)) (C-right . org-ref-next-key)
                         (C-left . org-ref-previous-key)
                         (16777337 lambda nil "Paste key at point. Assumes the first thing in the killring is a key." (interactive)
                          (org-ref-insert-key-at-point (car kill-ring)))
                         (16777303 lambda nil "Copy all the keys at point." (interactive)
                          (kill-new (org-element-property :path (org-element-context))))
                         (16777335 lambda nil (interactive) (kill-new (car (org-ref-get-bibtex-key-and-file))))
                         (16777318 lambda nil (interactive)
                          (save-excursion (org-ref-open-citation-at-point) (kill-new (org-ref-format-bibtex-entry-at-point))))
                         (16777319 . org-ref-google-scholar-at-point)
                         (16777317 lambda nil "Email entry at point" (interactive) (org-ref-open-citation-at-point)
                          (org-ref-email-bibtex-entry))
                         (16777315 . org-ref-wos-citing-at-point) (16777330 . org-ref-wos-related-at-point)
                         (16777326 . org-ref-open-notes-at-point) (16777328 . org-ref-open-pdf-at-point)
                         (16777333 . org-ref-open-url-at-point) (16777314 . org-ref-open-citation-at-point) (follow-link . mouse-face)
                         (mouse-3 . org-find-file-at-mouse) (mouse-2 . org-open-at-mouse))
                        )
                       ("Citep" :follow (lambda (_) (funcall org-ref-cite-onclick-function nil)) :export org-ref-format-Citep :complete
                        org-Citep-complete-link :help-echo
                        (lambda (window object position)
                         (when org-ref-show-citation-on-enter
                          (save-excursion (goto-char position)
                           (let ((s (org-ref-format-entry (org-ref-get-bibtex-key-under-cursor))))
                            (with-temp-buffer (insert s) (fill-paragraph) (buffer-string)))
                           )
                          )
                         )
                        :face org-ref-cite-link-face-fn :display full :keymap
                        (keymap (S-up . org-ref-sort-citation-link) (S-right lambda nil (interactive) (org-ref-swap-citation-link 1))
                         (S-left lambda nil (interactive) (org-ref-swap-citation-link -1)) (C-right . org-ref-next-key)
                         (C-left . org-ref-previous-key)
                         (16777337 lambda nil "Paste key at point. Assumes the first thing in the killring is a key." (interactive)
                          (org-ref-insert-key-at-point (car kill-ring)))
                         (16777303 lambda nil "Copy all the keys at point." (interactive)
                          (kill-new (org-element-property :path (org-element-context))))
                         (16777335 lambda nil (interactive) (kill-new (car (org-ref-get-bibtex-key-and-file))))
                         (16777318 lambda nil (interactive)
                          (save-excursion (org-ref-open-citation-at-point) (kill-new (org-ref-format-bibtex-entry-at-point))))
                         (16777319 . org-ref-google-scholar-at-point)
                         (16777317 lambda nil "Email entry at point" (interactive) (org-ref-open-citation-at-point)
                          (org-ref-email-bibtex-entry))
                         (16777315 . org-ref-wos-citing-at-point) (16777330 . org-ref-wos-related-at-point)
                         (16777326 . org-ref-open-notes-at-point) (16777328 . org-ref-open-pdf-at-point)
                         (16777333 . org-ref-open-url-at-point) (16777314 . org-ref-open-citation-at-point) (follow-link . mouse-face)
                         (mouse-3 . org-find-file-at-mouse) (mouse-2 . org-open-at-mouse))
                        )
                       ("Citet" :follow (lambda (_) (funcall org-ref-cite-onclick-function nil)) :export org-ref-format-Citet :complete
                        org-Citet-complete-link :help-echo
                        (lambda (window object position)
                         (when org-ref-show-citation-on-enter
                          (save-excursion (goto-char position)
                           (let ((s (org-ref-format-entry (org-ref-get-bibtex-key-under-cursor))))
                            (with-temp-buffer (insert s) (fill-paragraph) (buffer-string)))
                           )
                          )
                         )
                        :face org-ref-cite-link-face-fn :display full :keymap
                        (keymap (S-up . org-ref-sort-citation-link) (S-right lambda nil (interactive) (org-ref-swap-citation-link 1))
                         (S-left lambda nil (interactive) (org-ref-swap-citation-link -1)) (C-right . org-ref-next-key)
                         (C-left . org-ref-previous-key)
                         (16777337 lambda nil "Paste key at point. Assumes the first thing in the killring is a key." (interactive)
                          (org-ref-insert-key-at-point (car kill-ring)))
                         (16777303 lambda nil "Copy all the keys at point." (interactive)
                          (kill-new (org-element-property :path (org-element-context))))
                         (16777335 lambda nil (interactive) (kill-new (car (org-ref-get-bibtex-key-and-file))))
                         (16777318 lambda nil (interactive)
                          (save-excursion (org-ref-open-citation-at-point) (kill-new (org-ref-format-bibtex-entry-at-point))))
                         (16777319 . org-ref-google-scholar-at-point)
                         (16777317 lambda nil "Email entry at point" (interactive) (org-ref-open-citation-at-point)
                          (org-ref-email-bibtex-entry))
                         (16777315 . org-ref-wos-citing-at-point) (16777330 . org-ref-wos-related-at-point)
                         (16777326 . org-ref-open-notes-at-point) (16777328 . org-ref-open-pdf-at-point)
                         (16777333 . org-ref-open-url-at-point) (16777314 . org-ref-open-citation-at-point) (follow-link . mouse-face)
                         (mouse-3 . org-find-file-at-mouse) (mouse-2 . org-open-at-mouse))
                        )
                       ("citeyearpar" :follow (lambda (_) (funcall org-ref-cite-onclick-function nil)) :export org-ref-format-citeyearpar
                        :complete org-citeyearpar-complete-link :help-echo
                        (lambda (window object position)
                         (when org-ref-show-citation-on-enter
                          (save-excursion (goto-char position)
                           (let ((s (org-ref-format-entry (org-ref-get-bibtex-key-under-cursor))))
                            (with-temp-buffer (insert s) (fill-paragraph) (buffer-string)))
                           )
                          )
                         )
                        :face org-ref-cite-link-face-fn :display full :keymap
                        (keymap (S-up . org-ref-sort-citation-link) (S-right lambda nil (interactive) (org-ref-swap-citation-link 1))
                         (S-left lambda nil (interactive) (org-ref-swap-citation-link -1)) (C-right . org-ref-next-key)
                         (C-left . org-ref-previous-key)
                         (16777337 lambda nil "Paste key at point. Assumes the first thing in the killring is a key." (interactive)
                          (org-ref-insert-key-at-point (car kill-ring)))
                         (16777303 lambda nil "Copy all the keys at point." (interactive)
                          (kill-new (org-element-property :path (org-element-context))))
                         (16777335 lambda nil (interactive) (kill-new (car (org-ref-get-bibtex-key-and-file))))
                         (16777318 lambda nil (interactive)
                          (save-excursion (org-ref-open-citation-at-point) (kill-new (org-ref-format-bibtex-entry-at-point))))
                         (16777319 . org-ref-google-scholar-at-point)
                         (16777317 lambda nil "Email entry at point" (interactive) (org-ref-open-citation-at-point)
                          (org-ref-email-bibtex-entry))
                         (16777315 . org-ref-wos-citing-at-point) (16777330 . org-ref-wos-related-at-point)
                         (16777326 . org-ref-open-notes-at-point) (16777328 . org-ref-open-pdf-at-point)
                         (16777333 . org-ref-open-url-at-point) (16777314 . org-ref-open-citation-at-point) (follow-link . mouse-face)
                         (mouse-3 . org-find-file-at-mouse) (mouse-2 . org-open-at-mouse))
                        )
                       ("citeyear*" :follow (lambda (_) (funcall org-ref-cite-onclick-function nil)) :export org-ref-format-citeyear*
                        :complete org-citeyear*-complete-link :help-echo
                        (lambda (window object position)
                         (when org-ref-show-citation-on-enter
                          (save-excursion (goto-char position)
                           (let ((s (org-ref-format-entry (org-ref-get-bibtex-key-under-cursor))))
                            (with-temp-buffer (insert s) (fill-paragraph) (buffer-string)))
                           )
                          )
                         )
                        :face org-ref-cite-link-face-fn :display full :keymap
                        (keymap (S-up . org-ref-sort-citation-link) (S-right lambda nil (interactive) (org-ref-swap-citation-link 1))
                         (S-left lambda nil (interactive) (org-ref-swap-citation-link -1)) (C-right . org-ref-next-key)
                         (C-left . org-ref-previous-key)
                         (16777337 lambda nil "Paste key at point. Assumes the first thing in the killring is a key." (interactive)
                          (org-ref-insert-key-at-point (car kill-ring)))
                         (16777303 lambda nil "Copy all the keys at point." (interactive)
                          (kill-new (org-element-property :path (org-element-context))))
                         (16777335 lambda nil (interactive) (kill-new (car (org-ref-get-bibtex-key-and-file))))
                         (16777318 lambda nil (interactive)
                          (save-excursion (org-ref-open-citation-at-point) (kill-new (org-ref-format-bibtex-entry-at-point))))
                         (16777319 . org-ref-google-scholar-at-point)
                         (16777317 lambda nil "Email entry at point" (interactive) (org-ref-open-citation-at-point)
                          (org-ref-email-bibtex-entry))
                         (16777315 . org-ref-wos-citing-at-point) (16777330 . org-ref-wos-related-at-point)
                         (16777326 . org-ref-open-notes-at-point) (16777328 . org-ref-open-pdf-at-point)
                         (16777333 . org-ref-open-url-at-point) (16777314 . org-ref-open-citation-at-point) (follow-link . mouse-face)
                         (mouse-3 . org-find-file-at-mouse) (mouse-2 . org-open-at-mouse))
                        )
                       ("citeyear" :follow (lambda (_) (funcall org-ref-cite-onclick-function nil)) :export org-ref-format-citeyear :complete
                        org-citeyear-complete-link :help-echo
                        (lambda (window object position)
                         (when org-ref-show-citation-on-enter
                          (save-excursion (goto-char position)
                           (let ((s (org-ref-format-entry (org-ref-get-bibtex-key-under-cursor))))
                            (with-temp-buffer (insert s) (fill-paragraph) (buffer-string)))
                           )
                          )
                         )
                        :face org-ref-cite-link-face-fn :display full :keymap
                        (keymap (S-up . org-ref-sort-citation-link) (S-right lambda nil (interactive) (org-ref-swap-citation-link 1))
                         (S-left lambda nil (interactive) (org-ref-swap-citation-link -1)) (C-right . org-ref-next-key)
                         (C-left . org-ref-previous-key)
                         (16777337 lambda nil "Paste key at point. Assumes the first thing in the killring is a key." (interactive)
                          (org-ref-insert-key-at-point (car kill-ring)))
                         (16777303 lambda nil "Copy all the keys at point." (interactive)
                          (kill-new (org-element-property :path (org-element-context))))
                         (16777335 lambda nil (interactive) (kill-new (car (org-ref-get-bibtex-key-and-file))))
                         (16777318 lambda nil (interactive)
                          (save-excursion (org-ref-open-citation-at-point) (kill-new (org-ref-format-bibtex-entry-at-point))))
                         (16777319 . org-ref-google-scholar-at-point)
                         (16777317 lambda nil "Email entry at point" (interactive) (org-ref-open-citation-at-point)
                          (org-ref-email-bibtex-entry))
                         (16777315 . org-ref-wos-citing-at-point) (16777330 . org-ref-wos-related-at-point)
                         (16777326 . org-ref-open-notes-at-point) (16777328 . org-ref-open-pdf-at-point)
                         (16777333 . org-ref-open-url-at-point) (16777314 . org-ref-open-citation-at-point) (follow-link . mouse-face)
                         (mouse-3 . org-find-file-at-mouse) (mouse-2 . org-open-at-mouse))
                        )
                       ("citeauthor*" :follow (lambda (_) (funcall org-ref-cite-onclick-function nil)) :export org-ref-format-citeauthor*
                        :complete org-citeauthor*-complete-link :help-echo
                        (lambda (window object position)
                         (when org-ref-show-citation-on-enter
                          (save-excursion (goto-char position)
                           (let ((s (org-ref-format-entry (org-ref-get-bibtex-key-under-cursor))))
                            (with-temp-buffer (insert s) (fill-paragraph) (buffer-string)))
                           )
                          )
                         )
                        :face org-ref-cite-link-face-fn :display full :keymap
                        (keymap (S-up . org-ref-sort-citation-link) (S-right lambda nil (interactive) (org-ref-swap-citation-link 1))
                         (S-left lambda nil (interactive) (org-ref-swap-citation-link -1)) (C-right . org-ref-next-key)
                         (C-left . org-ref-previous-key)
                         (16777337 lambda nil "Paste key at point. Assumes the first thing in the killring is a key." (interactive)
                          (org-ref-insert-key-at-point (car kill-ring)))
                         (16777303 lambda nil "Copy all the keys at point." (interactive)
                          (kill-new (org-element-property :path (org-element-context))))
                         (16777335 lambda nil (interactive) (kill-new (car (org-ref-get-bibtex-key-and-file))))
                         (16777318 lambda nil (interactive)
                          (save-excursion (org-ref-open-citation-at-point) (kill-new (org-ref-format-bibtex-entry-at-point))))
                         (16777319 . org-ref-google-scholar-at-point)
                         (16777317 lambda nil "Email entry at point" (interactive) (org-ref-open-citation-at-point)
                          (org-ref-email-bibtex-entry))
                         (16777315 . org-ref-wos-citing-at-point) (16777330 . org-ref-wos-related-at-point)
                         (16777326 . org-ref-open-notes-at-point) (16777328 . org-ref-open-pdf-at-point)
                         (16777333 . org-ref-open-url-at-point) (16777314 . org-ref-open-citation-at-point) (follow-link . mouse-face)
                         (mouse-3 . org-find-file-at-mouse) (mouse-2 . org-open-at-mouse))
                        )
                       ("citeauthor" :follow (lambda (_) (funcall org-ref-cite-onclick-function nil)) :export org-ref-format-citeauthor
                        :complete org-citeauthor-complete-link :help-echo
                        (lambda (window object position)
                         (when org-ref-show-citation-on-enter
                          (save-excursion (goto-char position)
                           (let ((s (org-ref-format-entry (org-ref-get-bibtex-key-under-cursor))))
                            (with-temp-buffer (insert s) (fill-paragraph) (buffer-string)))
                           )
                          )
                         )
                        :face org-ref-cite-link-face-fn :display full :keymap
                        (keymap (S-up . org-ref-sort-citation-link) (S-right lambda nil (interactive) (org-ref-swap-citation-link 1))
                         (S-left lambda nil (interactive) (org-ref-swap-citation-link -1)) (C-right . org-ref-next-key)
                         (C-left . org-ref-previous-key)
                         (16777337 lambda nil "Paste key at point. Assumes the first thing in the killring is a key." (interactive)
                          (org-ref-insert-key-at-point (car kill-ring)))
                         (16777303 lambda nil "Copy all the keys at point." (interactive)
                          (kill-new (org-element-property :path (org-element-context))))
                         (16777335 lambda nil (interactive) (kill-new (car (org-ref-get-bibtex-key-and-file))))
                         (16777318 lambda nil (interactive)
                          (save-excursion (org-ref-open-citation-at-point) (kill-new (org-ref-format-bibtex-entry-at-point))))
                         (16777319 . org-ref-google-scholar-at-point)
                         (16777317 lambda nil "Email entry at point" (interactive) (org-ref-open-citation-at-point)
                          (org-ref-email-bibtex-entry))
                         (16777315 . org-ref-wos-citing-at-point) (16777330 . org-ref-wos-related-at-point)
                         (16777326 . org-ref-open-notes-at-point) (16777328 . org-ref-open-pdf-at-point)
                         (16777333 . org-ref-open-url-at-point) (16777314 . org-ref-open-citation-at-point) (follow-link . mouse-face)
                         (mouse-3 . org-find-file-at-mouse) (mouse-2 . org-open-at-mouse))
                        )
                       ("citetext" :follow (lambda (_) (funcall org-ref-cite-onclick-function nil)) :export org-ref-format-citetext :complete
                        org-citetext-complete-link :help-echo
                        (lambda (window object position)
                         (when org-ref-show-citation-on-enter
                          (save-excursion (goto-char position)
                           (let ((s (org-ref-format-entry (org-ref-get-bibtex-key-under-cursor))))
                            (with-temp-buffer (insert s) (fill-paragraph) (buffer-string)))
                           )
                          )
                         )
                        :face org-ref-cite-link-face-fn :display full :keymap
                        (keymap (S-up . org-ref-sort-citation-link) (S-right lambda nil (interactive) (org-ref-swap-citation-link 1))
                         (S-left lambda nil (interactive) (org-ref-swap-citation-link -1)) (C-right . org-ref-next-key)
                         (C-left . org-ref-previous-key)
                         (16777337 lambda nil "Paste key at point. Assumes the first thing in the killring is a key." (interactive)
                          (org-ref-insert-key-at-point (car kill-ring)))
                         (16777303 lambda nil "Copy all the keys at point." (interactive)
                          (kill-new (org-element-property :path (org-element-context))))
                         (16777335 lambda nil (interactive) (kill-new (car (org-ref-get-bibtex-key-and-file))))
                         (16777318 lambda nil (interactive)
                          (save-excursion (org-ref-open-citation-at-point) (kill-new (org-ref-format-bibtex-entry-at-point))))
                         (16777319 . org-ref-google-scholar-at-point)
                         (16777317 lambda nil "Email entry at point" (interactive) (org-ref-open-citation-at-point)
                          (org-ref-email-bibtex-entry))
                         (16777315 . org-ref-wos-citing-at-point) (16777330 . org-ref-wos-related-at-point)
                         (16777326 . org-ref-open-notes-at-point) (16777328 . org-ref-open-pdf-at-point)
                         (16777333 . org-ref-open-url-at-point) (16777314 . org-ref-open-citation-at-point) (follow-link . mouse-face)
                         (mouse-3 . org-find-file-at-mouse) (mouse-2 . org-open-at-mouse))
                        )
                       ("citenum" :follow (lambda (_) (funcall org-ref-cite-onclick-function nil)) :export org-ref-format-citenum :complete
                        org-citenum-complete-link :help-echo
                        (lambda (window object position)
                         (when org-ref-show-citation-on-enter
                          (save-excursion (goto-char position)
                           (let ((s (org-ref-format-entry (org-ref-get-bibtex-key-under-cursor))))
                            (with-temp-buffer (insert s) (fill-paragraph) (buffer-string)))
                           )
                          )
                         )
                        :face org-ref-cite-link-face-fn :display full :keymap
                        (keymap (S-up . org-ref-sort-citation-link) (S-right lambda nil (interactive) (org-ref-swap-citation-link 1))
                         (S-left lambda nil (interactive) (org-ref-swap-citation-link -1)) (C-right . org-ref-next-key)
                         (C-left . org-ref-previous-key)
                         (16777337 lambda nil "Paste key at point. Assumes the first thing in the killring is a key." (interactive)
                          (org-ref-insert-key-at-point (car kill-ring)))
                         (16777303 lambda nil "Copy all the keys at point." (interactive)
                          (kill-new (org-element-property :path (org-element-context))))
                         (16777335 lambda nil (interactive) (kill-new (car (org-ref-get-bibtex-key-and-file))))
                         (16777318 lambda nil (interactive)
                          (save-excursion (org-ref-open-citation-at-point) (kill-new (org-ref-format-bibtex-entry-at-point))))
                         (16777319 . org-ref-google-scholar-at-point)
                         (16777317 lambda nil "Email entry at point" (interactive) (org-ref-open-citation-at-point)
                          (org-ref-email-bibtex-entry))
                         (16777315 . org-ref-wos-citing-at-point) (16777330 . org-ref-wos-related-at-point)
                         (16777326 . org-ref-open-notes-at-point) (16777328 . org-ref-open-pdf-at-point)
                         (16777333 . org-ref-open-url-at-point) (16777314 . org-ref-open-citation-at-point) (follow-link . mouse-face)
                         (mouse-3 . org-find-file-at-mouse) (mouse-2 . org-open-at-mouse))
                        )
                       ("citealp*" :follow (lambda (_) (funcall org-ref-cite-onclick-function nil)) :export org-ref-format-citealp* :complete
                        org-citealp*-complete-link :help-echo
                        (lambda (window object position)
                         (when org-ref-show-citation-on-enter
                          (save-excursion (goto-char position)
                           (let ((s (org-ref-format-entry (org-ref-get-bibtex-key-under-cursor))))
                            (with-temp-buffer (insert s) (fill-paragraph) (buffer-string)))
                           )
                          )
                         )
                        :face org-ref-cite-link-face-fn :display full :keymap
                        (keymap (S-up . org-ref-sort-citation-link) (S-right lambda nil (interactive) (org-ref-swap-citation-link 1))
                         (S-left lambda nil (interactive) (org-ref-swap-citation-link -1)) (C-right . org-ref-next-key)
                         (C-left . org-ref-previous-key)
                         (16777337 lambda nil "Paste key at point. Assumes the first thing in the killring is a key." (interactive)
                          (org-ref-insert-key-at-point (car kill-ring)))
                         (16777303 lambda nil "Copy all the keys at point." (interactive)
                          (kill-new (org-element-property :path (org-element-context))))
                         (16777335 lambda nil (interactive) (kill-new (car (org-ref-get-bibtex-key-and-file))))
                         (16777318 lambda nil (interactive)
                          (save-excursion (org-ref-open-citation-at-point) (kill-new (org-ref-format-bibtex-entry-at-point))))
                         (16777319 . org-ref-google-scholar-at-point)
                         (16777317 lambda nil "Email entry at point" (interactive) (org-ref-open-citation-at-point)
                          (org-ref-email-bibtex-entry))
                         (16777315 . org-ref-wos-citing-at-point) (16777330 . org-ref-wos-related-at-point)
                         (16777326 . org-ref-open-notes-at-point) (16777328 . org-ref-open-pdf-at-point)
                         (16777333 . org-ref-open-url-at-point) (16777314 . org-ref-open-citation-at-point) (follow-link . mouse-face)
                         (mouse-3 . org-find-file-at-mouse) (mouse-2 . org-open-at-mouse))
                        )
                       ("citealp" :follow (lambda (_) (funcall org-ref-cite-onclick-function nil)) :export org-ref-format-citealp :complete
                        org-citealp-complete-link :help-echo
                        (lambda (window object position)
                         (when org-ref-show-citation-on-enter
                          (save-excursion (goto-char position)
                           (let ((s (org-ref-format-entry (org-ref-get-bibtex-key-under-cursor))))
                            (with-temp-buffer (insert s) (fill-paragraph) (buffer-string)))
                           )
                          )
                         )
                        :face org-ref-cite-link-face-fn :display full :keymap
                        (keymap (S-up . org-ref-sort-citation-link) (S-right lambda nil (interactive) (org-ref-swap-citation-link 1))
                         (S-left lambda nil (interactive) (org-ref-swap-citation-link -1)) (C-right . org-ref-next-key)
                         (C-left . org-ref-previous-key)
                         (16777337 lambda nil "Paste key at point. Assumes the first thing in the killring is a key." (interactive)
                          (org-ref-insert-key-at-point (car kill-ring)))
                         (16777303 lambda nil "Copy all the keys at point." (interactive)
                          (kill-new (org-element-property :path (org-element-context))))
                         (16777335 lambda nil (interactive) (kill-new (car (org-ref-get-bibtex-key-and-file))))
                         (16777318 lambda nil (interactive)
                          (save-excursion (org-ref-open-citation-at-point) (kill-new (org-ref-format-bibtex-entry-at-point))))
                         (16777319 . org-ref-google-scholar-at-point)
                         (16777317 lambda nil "Email entry at point" (interactive) (org-ref-open-citation-at-point)
                          (org-ref-email-bibtex-entry))
                         (16777315 . org-ref-wos-citing-at-point) (16777330 . org-ref-wos-related-at-point)
                         (16777326 . org-ref-open-notes-at-point) (16777328 . org-ref-open-pdf-at-point)
                         (16777333 . org-ref-open-url-at-point) (16777314 . org-ref-open-citation-at-point) (follow-link . mouse-face)
                         (mouse-3 . org-find-file-at-mouse) (mouse-2 . org-open-at-mouse))
                        )
                       ("citealt*" :follow (lambda (_) (funcall org-ref-cite-onclick-function nil)) :export org-ref-format-citealt* :complete
                        org-citealt*-complete-link :help-echo
                        (lambda (window object position)
                         (when org-ref-show-citation-on-enter
                          (save-excursion (goto-char position)
                           (let ((s (org-ref-format-entry (org-ref-get-bibtex-key-under-cursor))))
                            (with-temp-buffer (insert s) (fill-paragraph) (buffer-string)))
                           )
                          )
                         )
                        :face org-ref-cite-link-face-fn :display full :keymap
                        (keymap (S-up . org-ref-sort-citation-link) (S-right lambda nil (interactive) (org-ref-swap-citation-link 1))
                         (S-left lambda nil (interactive) (org-ref-swap-citation-link -1)) (C-right . org-ref-next-key)
                         (C-left . org-ref-previous-key)
                         (16777337 lambda nil "Paste key at point. Assumes the first thing in the killring is a key." (interactive)
                          (org-ref-insert-key-at-point (car kill-ring)))
                         (16777303 lambda nil "Copy all the keys at point." (interactive)
                          (kill-new (org-element-property :path (org-element-context))))
                         (16777335 lambda nil (interactive) (kill-new (car (org-ref-get-bibtex-key-and-file))))
                         (16777318 lambda nil (interactive)
                          (save-excursion (org-ref-open-citation-at-point) (kill-new (org-ref-format-bibtex-entry-at-point))))
                         (16777319 . org-ref-google-scholar-at-point)
                         (16777317 lambda nil "Email entry at point" (interactive) (org-ref-open-citation-at-point)
                          (org-ref-email-bibtex-entry))
                         (16777315 . org-ref-wos-citing-at-point) (16777330 . org-ref-wos-related-at-point)
                         (16777326 . org-ref-open-notes-at-point) (16777328 . org-ref-open-pdf-at-point)
                         (16777333 . org-ref-open-url-at-point) (16777314 . org-ref-open-citation-at-point) (follow-link . mouse-face)
                         (mouse-3 . org-find-file-at-mouse) (mouse-2 . org-open-at-mouse))
                        )
                       ("citealt" :follow (lambda (_) (funcall org-ref-cite-onclick-function nil)) :export org-ref-format-citealt :complete
                        org-citealt-complete-link :help-echo
                        (lambda (window object position)
                         (when org-ref-show-citation-on-enter
                          (save-excursion (goto-char position)
                           (let ((s (org-ref-format-entry (org-ref-get-bibtex-key-under-cursor))))
                            (with-temp-buffer (insert s) (fill-paragraph) (buffer-string)))
                           )
                          )
                         )
                        :face org-ref-cite-link-face-fn :display full :keymap
                        (keymap (S-up . org-ref-sort-citation-link) (S-right lambda nil (interactive) (org-ref-swap-citation-link 1))
                         (S-left lambda nil (interactive) (org-ref-swap-citation-link -1)) (C-right . org-ref-next-key)
                         (C-left . org-ref-previous-key)
                         (16777337 lambda nil "Paste key at point. Assumes the first thing in the killring is a key." (interactive)
                          (org-ref-insert-key-at-point (car kill-ring)))
                         (16777303 lambda nil "Copy all the keys at point." (interactive)
                          (kill-new (org-element-property :path (org-element-context))))
                         (16777335 lambda nil (interactive) (kill-new (car (org-ref-get-bibtex-key-and-file))))
                         (16777318 lambda nil (interactive)
                          (save-excursion (org-ref-open-citation-at-point) (kill-new (org-ref-format-bibtex-entry-at-point))))
                         (16777319 . org-ref-google-scholar-at-point)
                         (16777317 lambda nil "Email entry at point" (interactive) (org-ref-open-citation-at-point)
                          (org-ref-email-bibtex-entry))
                         (16777315 . org-ref-wos-citing-at-point) (16777330 . org-ref-wos-related-at-point)
                         (16777326 . org-ref-open-notes-at-point) (16777328 . org-ref-open-pdf-at-point)
                         (16777333 . org-ref-open-url-at-point) (16777314 . org-ref-open-citation-at-point) (follow-link . mouse-face)
                         (mouse-3 . org-find-file-at-mouse) (mouse-2 . org-open-at-mouse))
                        )
                       ("citep*" :follow (lambda (_) (funcall org-ref-cite-onclick-function nil)) :export org-ref-format-citep* :complete
                        org-citep*-complete-link :help-echo
                        (lambda (window object position)
                         (when org-ref-show-citation-on-enter
                          (save-excursion (goto-char position)
                           (let ((s (org-ref-format-entry (org-ref-get-bibtex-key-under-cursor))))
                            (with-temp-buffer (insert s) (fill-paragraph) (buffer-string)))
                           )
                          )
                         )
                        :face org-ref-cite-link-face-fn :display full :keymap
                        (keymap (S-up . org-ref-sort-citation-link) (S-right lambda nil (interactive) (org-ref-swap-citation-link 1))
                         (S-left lambda nil (interactive) (org-ref-swap-citation-link -1)) (C-right . org-ref-next-key)
                         (C-left . org-ref-previous-key)
                         (16777337 lambda nil "Paste key at point. Assumes the first thing in the killring is a key." (interactive)
                          (org-ref-insert-key-at-point (car kill-ring)))
                         (16777303 lambda nil "Copy all the keys at point." (interactive)
                          (kill-new (org-element-property :path (org-element-context))))
                         (16777335 lambda nil (interactive) (kill-new (car (org-ref-get-bibtex-key-and-file))))
                         (16777318 lambda nil (interactive)
                          (save-excursion (org-ref-open-citation-at-point) (kill-new (org-ref-format-bibtex-entry-at-point))))
                         (16777319 . org-ref-google-scholar-at-point)
                         (16777317 lambda nil "Email entry at point" (interactive) (org-ref-open-citation-at-point)
                          (org-ref-email-bibtex-entry))
                         (16777315 . org-ref-wos-citing-at-point) (16777330 . org-ref-wos-related-at-point)
                         (16777326 . org-ref-open-notes-at-point) (16777328 . org-ref-open-pdf-at-point)
                         (16777333 . org-ref-open-url-at-point) (16777314 . org-ref-open-citation-at-point) (follow-link . mouse-face)
                         (mouse-3 . org-find-file-at-mouse) (mouse-2 . org-open-at-mouse))
                        )
                       ("citep" :follow (lambda (_) (funcall org-ref-cite-onclick-function nil)) :export org-ref-format-citep :complete
                        org-citep-complete-link :help-echo
                        (lambda (window object position)
                         (when org-ref-show-citation-on-enter
                          (save-excursion (goto-char position)
                           (let ((s (org-ref-format-entry (org-ref-get-bibtex-key-under-cursor))))
                            (with-temp-buffer (insert s) (fill-paragraph) (buffer-string)))
                           )
                          )
                         )
                        :face org-ref-cite-link-face-fn :display full :keymap
                        (keymap (S-up . org-ref-sort-citation-link) (S-right lambda nil (interactive) (org-ref-swap-citation-link 1))
                         (S-left lambda nil (interactive) (org-ref-swap-citation-link -1)) (C-right . org-ref-next-key)
                         (C-left . org-ref-previous-key)
                         (16777337 lambda nil "Paste key at point. Assumes the first thing in the killring is a key." (interactive)
                          (org-ref-insert-key-at-point (car kill-ring)))
                         (16777303 lambda nil "Copy all the keys at point." (interactive)
                          (kill-new (org-element-property :path (org-element-context))))
                         (16777335 lambda nil (interactive) (kill-new (car (org-ref-get-bibtex-key-and-file))))
                         (16777318 lambda nil (interactive)
                          (save-excursion (org-ref-open-citation-at-point) (kill-new (org-ref-format-bibtex-entry-at-point))))
                         (16777319 . org-ref-google-scholar-at-point)
                         (16777317 lambda nil "Email entry at point" (interactive) (org-ref-open-citation-at-point)
                          (org-ref-email-bibtex-entry))
                         (16777315 . org-ref-wos-citing-at-point) (16777330 . org-ref-wos-related-at-point)
                         (16777326 . org-ref-open-notes-at-point) (16777328 . org-ref-open-pdf-at-point)
                         (16777333 . org-ref-open-url-at-point) (16777314 . org-ref-open-citation-at-point) (follow-link . mouse-face)
                         (mouse-3 . org-find-file-at-mouse) (mouse-2 . org-open-at-mouse))
                        )
                       ("citet*" :follow (lambda (_) (funcall org-ref-cite-onclick-function nil)) :export org-ref-format-citet* :complete
                        org-citet*-complete-link :help-echo
                        (lambda (window object position)
                         (when org-ref-show-citation-on-enter
                          (save-excursion (goto-char position)
                           (let ((s (org-ref-format-entry (org-ref-get-bibtex-key-under-cursor))))
                            (with-temp-buffer (insert s) (fill-paragraph) (buffer-string)))
                           )
                          )
                         )
                        :face org-ref-cite-link-face-fn :display full :keymap
                        (keymap (S-up . org-ref-sort-citation-link) (S-right lambda nil (interactive) (org-ref-swap-citation-link 1))
                         (S-left lambda nil (interactive) (org-ref-swap-citation-link -1)) (C-right . org-ref-next-key)
                         (C-left . org-ref-previous-key)
                         (16777337 lambda nil "Paste key at point. Assumes the first thing in the killring is a key." (interactive)
                          (org-ref-insert-key-at-point (car kill-ring)))
                         (16777303 lambda nil "Copy all the keys at point." (interactive)
                          (kill-new (org-element-property :path (org-element-context))))
                         (16777335 lambda nil (interactive) (kill-new (car (org-ref-get-bibtex-key-and-file))))
                         (16777318 lambda nil (interactive)
                          (save-excursion (org-ref-open-citation-at-point) (kill-new (org-ref-format-bibtex-entry-at-point))))
                         (16777319 . org-ref-google-scholar-at-point)
                         (16777317 lambda nil "Email entry at point" (interactive) (org-ref-open-citation-at-point)
                          (org-ref-email-bibtex-entry))
                         (16777315 . org-ref-wos-citing-at-point) (16777330 . org-ref-wos-related-at-point)
                         (16777326 . org-ref-open-notes-at-point) (16777328 . org-ref-open-pdf-at-point)
                         (16777333 . org-ref-open-url-at-point) (16777314 . org-ref-open-citation-at-point) (follow-link . mouse-face)
                         (mouse-3 . org-find-file-at-mouse) (mouse-2 . org-open-at-mouse))
                        )
                       ("citet" :follow (lambda (_) (funcall org-ref-cite-onclick-function nil)) :export org-ref-format-citet :complete
                        org-citet-complete-link :help-echo
                        (lambda (window object position)
                         (when org-ref-show-citation-on-enter
                          (save-excursion (goto-char position)
                           (let ((s (org-ref-format-entry (org-ref-get-bibtex-key-under-cursor))))
                            (with-temp-buffer (insert s) (fill-paragraph) (buffer-string)))
                           )
                          )
                         )
                        :face org-ref-cite-link-face-fn :display full :keymap
                        (keymap (S-up . org-ref-sort-citation-link) (S-right lambda nil (interactive) (org-ref-swap-citation-link 1))
                         (S-left lambda nil (interactive) (org-ref-swap-citation-link -1)) (C-right . org-ref-next-key)
                         (C-left . org-ref-previous-key)
                         (16777337 lambda nil "Paste key at point. Assumes the first thing in the killring is a key." (interactive)
                          (org-ref-insert-key-at-point (car kill-ring)))
                         (16777303 lambda nil "Copy all the keys at point." (interactive)
                          (kill-new (org-element-property :path (org-element-context))))
                         (16777335 lambda nil (interactive) (kill-new (car (org-ref-get-bibtex-key-and-file))))
                         (16777318 lambda nil (interactive)
                          (save-excursion (org-ref-open-citation-at-point) (kill-new (org-ref-format-bibtex-entry-at-point))))
                         (16777319 . org-ref-google-scholar-at-point)
                         (16777317 lambda nil "Email entry at point" (interactive) (org-ref-open-citation-at-point)
                          (org-ref-email-bibtex-entry))
                         (16777315 . org-ref-wos-citing-at-point) (16777330 . org-ref-wos-related-at-point)
                         (16777326 . org-ref-open-notes-at-point) (16777328 . org-ref-open-pdf-at-point)
                         (16777333 . org-ref-open-url-at-point) (16777314 . org-ref-open-citation-at-point) (follow-link . mouse-face)
                         (mouse-3 . org-find-file-at-mouse) (mouse-2 . org-open-at-mouse))
                        )
                       ("nocite" :follow (lambda (_) (funcall org-ref-cite-onclick-function nil)) :export org-ref-format-nocite :complete
                        org-nocite-complete-link :help-echo
                        (lambda (window object position)
                         (when org-ref-show-citation-on-enter
                          (save-excursion (goto-char position)
                           (let ((s (org-ref-format-entry (org-ref-get-bibtex-key-under-cursor))))
                            (with-temp-buffer (insert s) (fill-paragraph) (buffer-string)))
                           )
                          )
                         )
                        :face org-ref-cite-link-face-fn :display full :keymap
                        (keymap (S-up . org-ref-sort-citation-link) (S-right lambda nil (interactive) (org-ref-swap-citation-link 1))
                         (S-left lambda nil (interactive) (org-ref-swap-citation-link -1)) (C-right . org-ref-next-key)
                         (C-left . org-ref-previous-key)
                         (16777337 lambda nil "Paste key at point. Assumes the first thing in the killring is a key." (interactive)
                          (org-ref-insert-key-at-point (car kill-ring)))
                         (16777303 lambda nil "Copy all the keys at point." (interactive)
                          (kill-new (org-element-property :path (org-element-context))))
                         (16777335 lambda nil (interactive) (kill-new (car (org-ref-get-bibtex-key-and-file))))
                         (16777318 lambda nil (interactive)
                          (save-excursion (org-ref-open-citation-at-point) (kill-new (org-ref-format-bibtex-entry-at-point))))
                         (16777319 . org-ref-google-scholar-at-point)
                         (16777317 lambda nil "Email entry at point" (interactive) (org-ref-open-citation-at-point)
                          (org-ref-email-bibtex-entry))
                         (16777315 . org-ref-wos-citing-at-point) (16777330 . org-ref-wos-related-at-point)
                         (16777326 . org-ref-open-notes-at-point) (16777328 . org-ref-open-pdf-at-point)
                         (16777333 . org-ref-open-url-at-point) (16777314 . org-ref-open-citation-at-point) (follow-link . mouse-face)
                         (mouse-3 . org-find-file-at-mouse) (mouse-2 . org-open-at-mouse))
                        )
                       ("cite" :follow (lambda (_) (funcall org-ref-cite-onclick-function nil)) :export org-ref-format-cite :complete
                        org-cite-complete-link :help-echo
                        (lambda (window object position)
                         (when org-ref-show-citation-on-enter
                          (save-excursion (goto-char position)
                           (let ((s (org-ref-format-entry (org-ref-get-bibtex-key-under-cursor))))
                            (with-temp-buffer (insert s) (fill-paragraph) (buffer-string)))
                           )
                          )
                         )
                        :face org-ref-cite-link-face-fn :display full :keymap
                        (keymap (S-up . org-ref-sort-citation-link) (S-right lambda nil (interactive) (org-ref-swap-citation-link 1))
                         (S-left lambda nil (interactive) (org-ref-swap-citation-link -1)) (C-right . org-ref-next-key)
                         (C-left . org-ref-previous-key)
                         (16777337 lambda nil "Paste key at point. Assumes the first thing in the killring is a key." (interactive)
                          (org-ref-insert-key-at-point (car kill-ring)))
                         (16777303 lambda nil "Copy all the keys at point." (interactive)
                          (kill-new (org-element-property :path (org-element-context))))
                         (16777335 lambda nil (interactive) (kill-new (car (org-ref-get-bibtex-key-and-file))))
                         (16777318 lambda nil (interactive)
                          (save-excursion (org-ref-open-citation-at-point) (kill-new (org-ref-format-bibtex-entry-at-point))))
                         (16777319 . org-ref-google-scholar-at-point)
                         (16777317 lambda nil "Email entry at point" (interactive) (org-ref-open-citation-at-point)
                          (org-ref-email-bibtex-entry))
                         (16777315 . org-ref-wos-citing-at-point) (16777330 . org-ref-wos-related-at-point)
                         (16777326 . org-ref-open-notes-at-point) (16777328 . org-ref-open-pdf-at-point)
                         (16777333 . org-ref-open-url-at-point) (16777314 . org-ref-open-citation-at-point) (follow-link . mouse-face)
                         (mouse-3 . org-find-file-at-mouse) (mouse-2 . org-open-at-mouse))
                        :store org-ref-bibtex-store-link)
                       ("Cref" :follow org-ref-ref-follow :export org-ref-Cref-export :complete org-ref-complete-link :face
                        org-ref-ref-face-fn :help-echo org-ref-ref-help-echo)
                       ("cref" :follow org-ref-ref-follow :export org-ref-cref-export :complete org-ref-complete-link :face
                        org-ref-ref-face-fn :help-echo org-ref-ref-help-echo)
                       ("autoref" :follow org-ref-autoref-follow :export org-ref-autoref-export :complete org-ref-complete-link :face
                        org-ref-ref-face-fn :help-echo org-ref-ref-help-echo)
                       ("eqref" :follow org-ref-eqref-follow :export org-ref-eqref-export :complete org-ref-complete-link :face
                        org-ref-ref-face-fn :help-echo org-ref-ref-help-echo)
                       ("nameref" :follow org-ref-follow-nameref :export org-ref-export-nameref :complete org-ref-complete-link :face
                        org-ref-ref-face-fn :help-echo org-ref-ref-help-echo)
                       ("pageref" :follow org-ref-follow-pageref :export
                        #[(path desc format) "\b\302=\203\v\300\303	\"\207\b\304=\205\0\300\305	\"\207"
                          [format path html "(<pageref>%s</pageref>)" latex "\\pageref{%s}"] 3]
                        :face org-ref-ref-face-fn :complete org-pageref-complete-link :help-echo org-ref-ref-help-echo)
                       ("ref" :follow org-ref-ref-follow :export org-ref-ref-export :complete org-ref-complete-link :face org-ref-ref-face-fn
                        :help-echo org-ref-ref-help-echo)
                       ("label" :follow
                        #[(label) "\302\b!\303\304\305	\211\306U\204\0	\307V\203\0\310\202\0\311#\302\b!\")\207"
                          [label count org-ref-count-labels message format "%s occurence%s" 0 1 "s" ""] 6
                          "On clicking count the number of label tags used in the buffer.\nA number greater than one means multiple labels!"]
                        :export
                        #[(keyword desc format) "\b\302=\203\v\300\303	\"\207\b\304=\205\0\300\305	\"\207"
                          [format keyword html "<div id=\"%s\">" latex "\\label{%s}"] 3]
                        :store org-label-store-link :face org-ref-label-face-fn :help-echo
                        #[(window object position) "\212\bb\210\303 \304\305!r\nq\210\306\216	c\210\307 \210\310 -\207"
                          [position s temp-buffer org-ref-link-message generate-new-buffer " *temp*"
                           #[nil "\301\b!\205	\302\b!\207" [temp-buffer buffer-name kill-buffer] 2] fill-paragraph buffer-string]
                          2]
                        )
                       ("list-of-tables" :follow org-ref-list-of-tables :export
                        #[(keyword desc format) "\b\301=\205	\300\302!\207" [format latex "\\listoftables"] 2])
                       ("list-of-figures" :follow org-ref-list-of-figures :export
                        #[(keyword desc format) "\b\301=\205	\300\302!\207" [format latex "\\listoffigures"] 2])
                       ("addbibresource" :follow org-ref-follow-addbibresource :export
                        #[(keyword desc format) "\b\302=\203\n\300\303!\207\b\304=\205\0\300\305	\"\207"
                          [format keyword html "" latex "\\addbibresource{%s}"] 3]
                        )
                       ("bibliographystyle" :export
                        #[(keyword desc format) "\b\302=\204\f\b\303=\203\0\300\304	\"\207\305\207"
                          [format keyword latex beamer "\\bibliographystyle{%s}" ""] 3]
                        )
                       ("printbibliography" :follow org-ref-open-bibliography :export
                        #[(keyword desc format) "\b\301=\203	\302 \207\b\303=\203\0\304 \207\b\305=\205\0\306\207"
                          [format org org-ref-get-org-bibliography html org-ref-get-html-bibliography latex "\\printbibliography"] 2]
                        )
                       ("nobibliography" :follow org-ref-open-bibliography :export org-ref-nobibliography-format)
                       ("bibliography" :follow org-ref-open-bibliography :export org-ref-bibliography-format :complete
                        org-bibliography-complete-link :help-echo
                        #[(window object position) "\212\bb\210\303 \304\305!r\nq\210\306\216	c\210\307 \210\310 -\207"
                          [position s temp-buffer org-ref-link-message generate-new-buffer " *temp*"
                           #[nil "\301\b!\205	\302\b!\207" [temp-buffer buffer-name kill-buffer] 2] fill-paragraph buffer-string]
                          2]
                        :face org-ref-bibliography-face-fn)
                       ("Acp" :follow or-follow-acronym :face org-ref-acronym-face :help-echo or-acronym-tooltip :export
                        #[771 "\211\300=\203\v\301\302\"\207\301\303\226\"\207" [latex format "\\Glspl{%s}" "%s"] 6
                          "\n\n(fn PATH _ FORMAT)"]
                        )
                       ("acp" :follow or-follow-acronym :face org-ref-acronym-face :help-echo or-acronym-tooltip :export
                        #[771 "\211\300=\203\v\301\302\"\207\301\303\226\"\207" [latex format "\\glspl{%s}" "%s"] 6
                          "\n\n(fn PATH _ FORMAT)"]
                        )
                       ("Ac" :follow or-follow-acronym :face org-ref-acronym-face :help-echo or-acronym-tooltip :export
                        #[771 "\211\300=\203\v\301\302\"\207\301\303\226\"\207" [latex format "\\Gls{%s}" "%s"] 6
                          "\n\n(fn PATH _ FORMAT)"]
                        )
                       ("ac" :follow or-follow-acronym :face org-ref-acronym-face :help-echo or-acronym-tooltip :export
                        #[771 "\211\300=\203\v\301\302\"\207\301\303\226\"\207" [latex format "\\gls{%s}" "%s"] 6
                          "\n\n(fn PATH _ FORMAT)"]
                        )
                       ("acrfull" :follow or-follow-acronym :face org-ref-acronym-face :help-echo or-acronym-tooltip :export
                        #[771 "\211\300=\203\v\301\302\"\207\301\303\226\"\207" [latex format "\\acrfull{%s}" "%s"] 6
                          "\n\n(fn PATH _ FORMAT)"]
                        )
                       ("acrlong" :follow or-follow-acronym :face org-ref-acronym-face :help-echo or-acronym-tooltip :export
                        #[771 "\211\300=\203\v\301\302\"\207\301\303\226\"\207" [latex format "\\acrlong{%s}" "%s"] 6
                          "\n\n(fn PATH _ FORMAT)"]
                        )
                       ("acrshort" :follow or-follow-acronym :face org-ref-acronym-face :help-echo or-acronym-tooltip :export
                        #[771 "\211\300=\203\v\301\302\"\207\301\303\226\"\207" [latex format "\\acrshort{%s}" "%s"] 6
                          "\n\n(fn PATH _ FORMAT)"]
                        )
                       ("glsdesc" :follow or-follow-glossary :face org-ref-glossary-face :help-echo or-glossary-tooltip :export
                        #[771 "\211\300=\203\v\301\302\"\207\301\303\"\207" [latex format "\\glsdesc{%s}" "%s"] 6
                          "\n\n(fn PATH _ FORMAT)"]
                        )
                       ("glssymbol" :follow or-follow-glossary :face org-ref-glossary-face :help-echo or-glossary-tooltip :export
                        #[771 "\211\300=\203\v\301\302\"\207\301\303\"\207" [latex format "\\glssymbol{%s}" "%s"] 6
                          "\n\n(fn PATH DESC FORMAT)"]
                        )
                       ("glslink" :follow or-follow-glossary :face org-ref-glossary-face :help-echo or-glossary-tooltip :export
                        #[771 "\211\300=\203\f\301\302\x04#\207\301\303\"\207" [latex format "\\glslink{%s}{%s}" "%s"] 7
                          "\n\n(fn PATH DESC FORMAT)"]
                        )
                       ("Glspl" :follow or-follow-glossary :face org-ref-glossary-face :help-echo or-glossary-tooltip :export
                        #[771 "\211\300=\203\v\301\302\"\207\301\303\"\207" [latex format "\\Glspl{%s}" "%s"] 6 "\n\n(fn PATH _ FORMAT)"]
                        )
                       ("Gls" :follow or-follow-glossary :face org-ref-glossary-face :help-echo or-glossary-tooltip :export
                        #[771 "\211\300=\203\v\301\302\"\207\301\303\"\207" [latex format "\\Gls{%s}" "%s"] 6 "\n\n(fn PATH _ FORMAT)"])
                       ("glspl" :follow or-follow-glossary :face org-ref-glossary-face :help-echo or-glossary-tooltip :export
                        #[771 "\211\300=\203\v\301\302\"\207\301\303\"\207" [latex format "\\glspl{%s}" "%s"] 6 "\n\n(fn PATH _ FORMAT)"]
                        )
                       ("gls" :follow or-follow-glossary :face org-ref-glossary-face :help-echo or-glossary-tooltip :export
                        #[771 "\211\300=\203\v\301\302\"\207\301\303\"\207" [latex format "\\gls{%s}" "%s"] 6 "\n\n(fn PATH _ FORMAT)"])
                       ("bibtex" :follow org-bibtex-open :store org-bibtex-store-link) ("id" :follow org-id-open) ("file+sys") ("file+emacs")
                       ("doi" :follow doi-link-menu :export
                        #[(doi desc format)
                          "\b\304=\203\0\300\305	\n\v\206\0\306\nP$\207\b\307=\205%\300\310	\n\v\206$\306\nP$\207"
                          [format doi-utils-dx-doi-org-url doi desc html "<a href=\"%s%s\">%s</a>" "doi:" latex "\\href{%s%s}{%s}"] 6]
                        )
                       ("elisp" :follow org--open-elisp-link) ("file" :complete org-file-complete-link)
                       ("ftp" :follow (lambda (path) (browse-url (concat "ftp:" path)))) ("help" :follow org--open-help-link)
                       ("http" :follow (lambda (path) (browse-url (concat "http:" path))))
                       ("https" :follow (lambda (path) (browse-url (concat "https:" path))))
                       ("mailto" :follow (lambda (path) (browse-url (concat "mailto:" path))))
                       ("news" :follow (lambda (path) (browse-url (concat "news:" path)))) ("shell" :follow org--open-shell-link))
 org-latex-format-headline-function 'org-latex-format-headline-default-function
 org-capture-templates '(("m" "Mail options")
                         ("me" "Est+Num" entry (file+headline "~/ALLES/HGs/tex/vorlesungen/HGQuim-Num-Estad/Hojas.org" "Examen")
                          "* %T %^{Title} : %:subject %^G\n- From :: %:from\n- Subject :: %:subject\n- Date :: %:date\n- Email :: %a\n\n%?\n%i")
                         ("mi" "mailimportant" entry (file+headline "~/ALLES/Mail/README.org" "Important Mail")
                          "* %T %^{Title} : %:subject %^G\n- From :: %:from\n- Subject :: %:subject\n- Date :: %:date\n- Email :: %a\n\n%?\n%i")
                         ("ml" "maillavi" entry (file+headline "~/ALLES/HGs/Karp/Pub/Letter.org" "To the Editor")
                          "* %T %^{Title} : %:subject %^G\n- From :: %:from\n- Subject :: %:subject\n- Date :: %:date\n- Email :: %a\n\n%?\n%i")
                         ("mr" "MailMathRev" entry (file+headline "~/ALLES/MathRev/ToDo/TODO.org" "To the Editor")
                          "* %T TODO %^{Task} : %:subject %^G\nSCHEDULED: %t\n- From :: %:from\n- Subject :: %:subject\n- Date :: %:date\n- Email :: %a\n\n%?\n%i" :kill-buffer t)
                         ("r" "Reply" entry (file+headline "~/Mail/TODO-email.org" "Emails") "** %T REPLY %a %?" :gnus-attachments t)
                         ("l" "LaTeX(headline) Generic README.org" entry
                          (file+headline (lambda nil (expand-file-name (read-file-name "Name of file (curr dir;headline-format format)!: ")))
                           "Overview")
                          "* TODO %?, %^g %^{prompt} Link: %A\n\n %i")
                         ("c" "Contacts" entry (file "~/ALLES/contacts.org")
                          "* %(org-contacts-template-name)\n:PROPERTIES:\n:EMAIL: %(org-contacts-template-email)\n:PHONE:\n:ALIAS:\n:NICKNAME:\n:IGNORE:\n:ICON:\n:NOTE:\n:ADDRESS:\n:BIRTHDAY:\n:END:")
                         ("j" "Journal Entry" entry (file+olp+datetree "~/ALLES/journal.org") "* %?" :empty-lines 1)
                         ("G" "Generic README.org file (table format)" table-line
                          (file+headline (lambda nil (expand-file-name (read-file-name "Name of file (curr dir;table format)!: "))) "Overview")
                          "|%f| %U|%A|" :prepend t)
                         ("w" "Web site" entry (file "/home/oub/ALLES/download.org") "* %a :website:\n\n%U %?\n\n%:initial"))
 org-agenda-file-regexp "\\`[^.].*\\.org\\|.todo\\'"
 org-latex-format-inlinetask-function 'org-latex-format-inlinetask-default-function
 org-tab-before-tab-emulation-hook '(org-tempo-complete-tag)
 org-html-format-drawer-function #[514 "\207" [] 3 "\n\n(fn NAME CONTENTS)"]
 outline-mode-hook '(hide-body)
 org-latex-packages-alist '(("numbered,framed" "matlab-prettifier" t))
 org-image-actual-width 400
 org-display-custom-times t
 org-export-with-toc nil
 org-ref-get-pdf-filename-function 'org-ref-get-pdf-filename
 org-latex-classes '(("article" "\\documentclass[12pt]{article}" ("\\section{%s}" . "\\section*{%s}")
                      ("\\subsection{%s}" . "\\subsection*{%s}") ("\\subsubsection{%s}" . "\\subsubsection*{%s}")
                      ("\\paragraph{%s}" . "\\paragraph*{%s}") ("\\subparagraph{%s}" . "\\subparagraph*{%s}"))
                     ("report" "\\documentclass[12pt]{report}" ("\\part{%s}" . "\\part*{%s}") ("\\chapter{%s}" . "\\chapter*{%s}")
                      ("\\section{%s}" . "\\section*{%s}") ("\\subsection{%s}" . "\\subsection*{%s}")
                      ("\\subsubsection{%s}" . "\\subsubsection*{%s}"))
                     ("book" "\\documentclass[12pt]{book}" ("\\part{%s}" . "\\part*{%s}") ("\\chapter{%s}" . "\\chapter*{%s}")
                      ("\\section{%s}" . "\\section*{%s}") ("\\subsection{%s}" . "\\subsection*{%s}")
                      ("\\subsubsection{%s}" . "\\subsubsection*{%s}"))
                     ("exam" "\\documentclass[12pt, addpoints, answers]{exam}"
                      ("\\begin{questions}%%%s" "\\end{questions}" "\\begin{questions}%%%s" "\\end{questions}") ("\\titledquestion{%s}" . "")
                      ("\\begin{parts}%%%s" "\\end{parts}" "\\begin{parts}%%%s" "\\end{parts}") ("\\part[%s] " . "")
                      ("\\begin{subparts}%%%s" "\\end{subparts}" "\\begin{subparts}%%%s" "\\end{subparts}") ("\\subpart[%s] " . "")
                      ("\\begin{solution}[%s]" "\\end{solution}" "\\begin{solution}[%s]" "\\end{solution}"))
                     )
 org-html-format-headline-function 'org-html-format-headline-default-function
 org-entities-user '(("implies" "\\Rightarrow" t "&rArr;" "=>" "=>" "⇒") ("iff" "\\iff" t "&hArr;" "<=>" "<=>" "⇔"))
 org-ref-notes-function '(lambda (thekey)
                          (let* ((results (org-ref-get-bibtex-key-and-file thekey)) (key (car results)) (bibfile (cdr results)))
                           (save-excursion
                            (with-temp-buffer (insert-file-contents bibfile) (bibtex-set-dialect (parsebib-find-bibtex-dialect) t)
                             (bibtex-search-entry key) (org-ref-open-bibtex-notes))
                            )
                           )
                          )
 org-latex-listings-options '(("style" "Matlab-editor") ("basicstyle" "\\mlttfamily") ("escapechar" "\"") ("mlshowsectionrules" "true")
                              ("mathescape" "true") ("morecomment" "[s]{\\%\\{}{\\%\\}}"))
 org-odt-convert-processes '(("gnumeric" "/usr/bin/ssconvert %i %o"))
 org-list-allow-alphabetical t
 )

^ permalink raw reply	[flat|nested] 5+ messages in thread

* Re: Bug: cannot export to beamer, even with (require 'ox-beamer) [9.1.14 (release_9.1.14-1059-gadec50 @ /home/oub/emacs/site-lisp/packages/org/)]
  2018-12-17  8:07 Bug: cannot export to beamer, even with (require 'ox-beamer) [9.1.14 (release_9.1.14-1059-gadec50 @ /home/oub/emacs/site-lisp/packages/org/)] Uwe Brauer
@ 2018-12-17 18:07 ` Nick Dokos
  2018-12-17 19:54   ` Uwe Brauer
  0 siblings, 1 reply; 5+ messages in thread
From: Nick Dokos @ 2018-12-17 18:07 UTC (permalink / raw)
  To: emacs-orgmode

Uwe Brauer <oub@mat.ucm.es> writes:

> I have  
> (require 'ox-beamer) 
>
> In my org init file, which should via add-to-list, add beamer but somehow
> it does not. 
>
> So when I call 
>
> C-c C-e l b
>
> In a org file I obtain the message
>
> user-error: Unknown LaTeX class ‘beamer’
>
> Something is wrong here. Add-to-list should add beamer to the org-class
> but somehow it does not.
> I am not sure who is to blame. Org or customize?
>

You are probably loading ox-beamer which adds it and then loading the
customized version which resets it. Try changing the order.

-- 
Nick

"There are only two hard problems in computer science: cache
invalidation, naming things, and off-by-one errors." -Martin Fowler

^ permalink raw reply	[flat|nested] 5+ messages in thread

* Re: Bug: cannot export to beamer, even with (require 'ox-beamer) [9.1.14 (release_9.1.14-1059-gadec50 @ /home/oub/emacs/site-lisp/packages/org/)]
  2018-12-17 18:07 ` Nick Dokos
@ 2018-12-17 19:54   ` Uwe Brauer
  2018-12-18  3:22     ` Nick Dokos
  0 siblings, 1 reply; 5+ messages in thread
From: Uwe Brauer @ 2018-12-17 19:54 UTC (permalink / raw)
  To: emacs-orgmode


[-- Attachment #1.1: Type: text/plain, Size: 813 bytes --]


> Uwe Brauer <oub@mat.ucm.es> writes:

> You are probably loading ox-beamer which adds it and then loading the
> customized version which resets it. Try changing the order.

The setting in my .emacs is:
(load-file "~/emacs/init/emacs_init.el")
(setq custom-file "/home/oub/emacs/init/custom-init.el")
(load-file "/home/oub/emacs/init/custom-init.el") 



So I changed that to
       (setq custom-file "/home/oub/emacs/init/custom-init.el")
        (load-file "/home/oub/emacs/init/custom-init.el") 
        (load-file "~/emacs/init/emacs_init.el")

Then the export works, but other problems occur. If I open a org file 

I receive a error message I attach, and indeed in that org file my
settings as defined in org_init are not all recognized.
Very strange indeed

Here is the error



[-- Attachment #1.2: org-bug.txt --]
[-- Type: text/plain, Size: 6196 bytes --]

Debugger entered--Lisp error: (void-function org--check-org-structure-template-alist)
  org--check-org-structure-template-alist(org-structure-template-alist)
  mapc(org--check-org-structure-template-alist (org-structure-template-alist org-tempo-keywords-alist))
  org-tempo-add-templates()
  org-tempo--update-maybe()
  org-tempo-setup()
  run-hooks(change-major-mode-after-body-hook text-mode-hook outline-mode-hook org-mode-hook)
  apply(run-hooks (change-major-mode-after-body-hook text-mode-hook outline-mode-hook org-mode-hook))
  run-mode-hooks(org-mode-hook)
  org-mode()
  set-auto-mode-0(org-mode nil)
  set-auto-mode()
  normal-mode(t)
  after-find-file(nil t)
  find-file-noselect-1(#<buffer beamer-exp.org> "~/ALLES/HGs/tex/vorlesungen/HGMetNum/Practicas/Prac5/Org-files/beamer-exp.org" nil nil "~/ALLES/HGs/tex/vorlesungen/HGMetNum/Practicas/Prac5/Org-files/beamer-exp.org" (9324866 2050))
  find-file-noselect("/home/oub/ALLES/HGs/tex/vorlesungen/HGMetNum/Practicas/Prac5/Org-files/beamer-exp.org" nil nil nil)
  #f(compiled-function (filename &optional wildcards) "Edit file FILENAME.\nSwitch to a buffer visiting file FILENAME,\ncreating one if none already exists.\nInteractively, the default if you just type RET is the current directory,\nbut the visited file name is available through the minibuffer history:\ntype \\[next-history-element] to pull it into the minibuffer.\n\nThe first time \\[next-history-element] is used after Emacs prompts for\nthe file name, the result is affected by `file-name-at-point-functions',\nwhich by default try to guess the file name by looking at point in the\ncurrent buffer.  Customize the value of `file-name-at-point-functions'\nor set it to nil, if you want only the visited file name and the\ncurrent directory to be available on first \\[next-history-element]\nrequest.\n\nYou can visit files on remote machines by specifying something\nlike /ssh:SOME_REMOTE_MACHINE:FILE for the file name.  You can\nalso visit local files as a different user by specifying\n/sudo::FILE for the file name.\nSee the Info node `(tramp)File name Syntax' in the Tramp Info\nmanual, for more about this.\n\nInteractively, or if WILDCARDS is non-nil in a call from Lisp,\nexpand wildcards (if any) and visit multiple files.  You can\nsuppress wildcard expansion by setting `find-file-wildcards' to nil.\n\nTo visit a file without any kind of conversion and without\nautomatically choosing a major mode, use \\[find-file-literally]." (interactive #f(compiled-function () #<bytecode 0x342cc75>)) #<bytecode 0x232d53>)("/home/oub/ALLES/HGs/tex/vorlesungen/HGMetNum/Practicas/Prac5/Org-files/beamer-exp.org" nil)
  ad-Advice-find-file(#f(compiled-function (filename &optional wildcards) "Edit file FILENAME.\nSwitch to a buffer visiting file FILENAME,\ncreating one if none already exists.\nInteractively, the default if you just type RET is the current directory,\nbut the visited file name is available through the minibuffer history:\ntype \\[next-history-element] to pull it into the minibuffer.\n\nThe first time \\[next-history-element] is used after Emacs prompts for\nthe file name, the result is affected by `file-name-at-point-functions',\nwhich by default try to guess the file name by looking at point in the\ncurrent buffer.  Customize the value of `file-name-at-point-functions'\nor set it to nil, if you want only the visited file name and the\ncurrent directory to be available on first \\[next-history-element]\nrequest.\n\nYou can visit files on remote machines by specifying something\nlike /ssh:SOME_REMOTE_MACHINE:FILE for the file name.  You can\nalso visit local files as a different user by specifying\n/sudo::FILE for the file name.\nSee the Info node `(tramp)File name Syntax' in the Tramp Info\nmanual, for more about this.\n\nInteractively, or if WILDCARDS is non-nil in a call from Lisp,\nexpand wildcards (if any) and visit multiple files.  You can\nsuppress wildcard expansion by setting `find-file-wildcards' to nil.\n\nTo visit a file without any kind of conversion and without\nautomatically choosing a major mode, use \\[find-file-literally]." (interactive #f(compiled-function () #<bytecode 0x342cdb5>)) #<bytecode 0x232d53>) "/home/oub/ALLES/HGs/tex/vorlesungen/HGMetNum/Practicas/Prac5/Org-files/beamer-exp.org")
  apply(ad-Advice-find-file #f(compiled-function (filename &optional wildcards) "Edit file FILENAME.\nSwitch to a buffer visiting file FILENAME,\ncreating one if none already exists.\nInteractively, the default if you just type RET is the current directory,\nbut the visited file name is available through the minibuffer history:\ntype \\[next-history-element] to pull it into the minibuffer.\n\nThe first time \\[next-history-element] is used after Emacs prompts for\nthe file name, the result is affected by `file-name-at-point-functions',\nwhich by default try to guess the file name by looking at point in the\ncurrent buffer.  Customize the value of `file-name-at-point-functions'\nor set it to nil, if you want only the visited file name and the\ncurrent directory to be available on first \\[next-history-element]\nrequest.\n\nYou can visit files on remote machines by specifying something\nlike /ssh:SOME_REMOTE_MACHINE:FILE for the file name.  You can\nalso visit local files as a different user by specifying\n/sudo::FILE for the file name.\nSee the Info node `(tramp)File name Syntax' in the Tramp Info\nmanual, for more about this.\n\nInteractively, or if WILDCARDS is non-nil in a call from Lisp,\nexpand wildcards (if any) and visit multiple files.  You can\nsuppress wildcard expansion by setting `find-file-wildcards' to nil.\n\nTo visit a file without any kind of conversion and without\nautomatically choosing a major mode, use \\[find-file-literally]." (interactive #f(compiled-function () #<bytecode 0x3190505>)) #<bytecode 0x232d53>) "/home/oub/ALLES/HGs/tex/vorlesungen/HGMetNum/Practicas/Prac5/Org-files/beamer-exp.org")
  find-file("/home/oub/ALLES/HGs/tex/vorlesungen/HGMetNum/Practicas/Prac5/Org-files/beamer-exp.org")
  dired-find-file()
  funcall-interactively(dired-find-file)
  call-interactively(dired-find-file nil nil)
  command-execute(dired-find-file)

[-- Attachment #2: smime.p7s --]
[-- Type: application/pkcs7-signature, Size: 5025 bytes --]

^ permalink raw reply	[flat|nested] 5+ messages in thread

* Re: Bug: cannot export to beamer, even with (require 'ox-beamer) [9.1.14 (release_9.1.14-1059-gadec50 @ /home/oub/emacs/site-lisp/packages/org/)]
  2018-12-17 19:54   ` Uwe Brauer
@ 2018-12-18  3:22     ` Nick Dokos
  2018-12-18  8:13       ` Uwe Brauer
  0 siblings, 1 reply; 5+ messages in thread
From: Nick Dokos @ 2018-12-18  3:22 UTC (permalink / raw)
  To: emacs-orgmode

Uwe Brauer <oub@mat.ucm.es> writes:

>> Uwe Brauer <oub@mat.ucm.es> writes:
>
>> You are probably loading ox-beamer which adds it and then loading the
>> customized version which resets it. Try changing the order.
>
> The setting in my .emacs is:
> (load-file "~/emacs/init/emacs_init.el")
> (setq custom-file "/home/oub/emacs/init/custom-init.el")
> (load-file "/home/oub/emacs/init/custom-init.el") 
>
>
>
> So I changed that to
>        (setq custom-file "/home/oub/emacs/init/custom-init.el")
>         (load-file "/home/oub/emacs/init/custom-init.el") 
>         (load-file "~/emacs/init/emacs_init.el")
>
> Then the export works, but other problems occur. If I open a org file 
>

If your only problem with the original order was what you described before,
don't change the order: just do

(load-file "~/emacs/init/emacs_init.el")
(setq custom-file "/home/oub/emacs/init/custom-init.el")
(load-file "/home/oub/emacs/init/custom-init.el")
(require 'ox-beamer)

and delete the original `(require 'ox-beamer)' from wherever it was
before (presumably in ~/emacs/init/emacs_init.el). You are only trying
to avoid the loading of your customizations from clobbering
org-latex-classes.

BTW, the fact that you *do* have problems when you change the order,
probably means that there is strong coupling between your init files
which is not a good thing. You might want to spend some time
refactoring them and breaking interdependencies. If you decide to do
that, small steps (and good backups) are probably a good idea.

-- 
Nick

"There are only two hard problems in computer science: cache
invalidation, naming things, and off-by-one errors." -Martin Fowler

^ permalink raw reply	[flat|nested] 5+ messages in thread

* Re: Bug: cannot export to beamer, even with (require 'ox-beamer) [9.1.14 (release_9.1.14-1059-gadec50 @ /home/oub/emacs/site-lisp/packages/org/)]
  2018-12-18  3:22     ` Nick Dokos
@ 2018-12-18  8:13       ` Uwe Brauer
  0 siblings, 0 replies; 5+ messages in thread
From: Uwe Brauer @ 2018-12-18  8:13 UTC (permalink / raw)
  To: emacs-orgmode

[-- Attachment #1: Type: text/plain, Size: 1023 bytes --]


   > Uwe Brauer <oub@mat.ucm.es> writes:

   > If your only problem with the original order was what you described before,
   > don't change the order: just do

   > (load-file "~/emacs/init/emacs_init.el")
   > (setq custom-file "/home/oub/emacs/init/custom-init.el")
   > (load-file "/home/oub/emacs/init/custom-init.el")
   > (require 'ox-beamer)

   > and delete the original `(require 'ox-beamer)' from wherever it was
   > before (presumably in ~/emacs/init/emacs_init.el). You are only trying
   > to avoid the loading of your customizations from clobbering
   > org-latex-classes.

   > BTW, the fact that you *do* have problems when you change the order,
   > probably means that there is strong coupling between your init files
   > which is not a good thing. You might want to spend some time
   > refactoring them and breaking interdependencies. If you decide to do
   > that, small steps (and good backups) are probably a good idea.

Right, that is the feeling I am starting to share. I might do that. thanks

[-- Attachment #2: smime.p7s --]
[-- Type: application/pkcs7-signature, Size: 4393 bytes --]

^ permalink raw reply	[flat|nested] 5+ messages in thread

end of thread, other threads:[~2018-12-18  8:13 UTC | newest]

Thread overview: 5+ messages (download: mbox.gz / follow: Atom feed)
-- links below jump to the message on this page --
2018-12-17  8:07 Bug: cannot export to beamer, even with (require 'ox-beamer) [9.1.14 (release_9.1.14-1059-gadec50 @ /home/oub/emacs/site-lisp/packages/org/)] Uwe Brauer
2018-12-17 18:07 ` Nick Dokos
2018-12-17 19:54   ` Uwe Brauer
2018-12-18  3:22     ` Nick Dokos
2018-12-18  8:13       ` Uwe Brauer

Code repositories for project(s) associated with this public inbox

	https://git.savannah.gnu.org/cgit/emacs/org-mode.git

This is a public inbox, see mirroring instructions
for how to clone and mirror all data and code used for this inbox;
as well as URLs for read-only IMAP folder(s) and NNTP newsgroup(s).